OB Chap 21 Nursing Management of Labor and Birth at Risk

अब Quizwiz के साथ अपने होमवर्क और परीक्षाओं को एस करें!

A patient is 32 weeks gestation and sent home on modified bedrest for preterm labor. She is on tocolyitics and wants to know when she can have intercourse again with her husband. What is the most appropriate response by the nurse? a) "You will not be able to have intercourse again until 6 weeks after you deliver." b) "The need to keep the infant safe should be of more concern than when to have sex." c) "That is a question to ask your health care provider, at this point you are on pelvic rest to try and stop any further labor." d) "Intercourse has nothing to do with preterm labor; you can have sex with your husband."

"That is a question to ask your health care provider, at this point you are on pelvic rest to try and stop any further labor."

A client is 32 weeks pregnant and sent home on modified bedrest for preterm labor. She is on tocolytics and wants to know when she can have intercourse again with her husband. What is the most appropriate response by the nurse?

"That is a question to ask your health care provider; at this point you are on pelvic rest to try and stop any further labor." The client needs to be on pelvic rest until the health care provider says otherwise. The intercourse can cause excitability in the uterus and encourage cervical softening and should be avoided unless the provider says it is safe.

During labor, a woman at 41 weeks gestation notes her amniotic fluid is leaking and is green in color. She is asking the nurse why the fluid is green. What is an appropriate response by the nurse? a) "Green might be a yeast infection and we need to culture the discharge." b) "You have an infection and need antibiotics." c) "This is meconium-stained fluid from the baby." d) "Amniotic fluid is normally green."

"This is meconium-stained fluid from the baby."

The experienced labor and delivery nurse knows to evaluate progress in active labor by using which simple rule? a) 1/4 cm/hour for cervical dilation b) 1 cm/hour for cervical dilation c) 1/2 cm/hour for cervical dilation d) 2 cm/hour for cervical dilation

1 cm/hour for cervical dilation

The experienced labor and birth nurse knows to evaluate progress in active labor by using which simple rule?

1 cm/hour for cervical dilation In evaluating the progress in active labor, the nurse uses the simple rule of 1 cm/hour for cervical dilation.

The nurse is caring for a client after experiencing a placental abruption. Which finding is the priority to report to the health care provider?

45 ml urine output in 2 hours The nurse knows a placental abruption places the client at high risk of hemorrhage. A decreased urine output indicates decreased perfusion from blood loss. The hematocrit, hemoglobin, and platelet counts are all within expected levels.

A woman is to undergo labor induction. The nurse determines that the woman most likely requires cervical ripening if her Bishop score is: a) 6 b) 9 c) 7 d) 5

5

A woman is to undergo labor induction. The nurse determines that the woman most likely requires cervical ripening if her Bishop score is: a) 6 b) 9 c) 5 d) 7

5 Correct Explanation: A Bishop score less than 6 usually indicates that a cervical ripening method should be used before labor induction.

Why is it important for the nurse to thoroughly assess maternal bladder and bowel status during labor? a) If the woman's bladder is distended, it may rupture. b) A full bladder or rectum can impede fetal descent. c) If the woman has a full bladder, labor may be uncomfortable for her. d) A full rectum can cause diarrhea.

A full bladder or rectum can impede fetal descent.

A nurse is assigned to care for a client who has to undergo a forceps and vacuum-assisted birth. The nurse understands that which of the following factors has contributed to a forceps and vacuum-assisted birth? a) Rupture of uterus b) Preterm labor with premature rupture of membranes c) Oligohydramnios due to placental insufficiency d) A prolonged second stage of labor

A prolonged second stage of labor

A woman is going to have labor induced with oxytocin. Which statement below reflects the induction technique you anticipate her primary-care provider will order? a) Administer Pitocin in two divided intramuscular sites. b) Administer oxytocin diluted as a "piggyback" infusion. c) Administer Pitocin in a 20 cc bolus of saline. d) Administer oxytocin diluted in the main intravenous fluid.

Administer oxytocin diluted as a "piggyback" infusion.

Immediately after delivering a full-term infant, a patient develops dyspnea and cyanosis. Her blood pressure decreases to 60/40 mm Hg, and she becomes unresponsive. What does the nurse suspect is happening with this patient? a) Congestive heart failure b) Placental separation c) Amniotic fluid embolism d) Aspiration

Amniotic fluid embolism

Immediately after delivering a full-term infant, a patient develops dyspnea and cyanosis. Her blood pressure decreases to 60/40 mm Hg, and she becomes unresponsive. What does the nurse suspect is happening with this patient? a) Congestive heart failure b) Amniotic fluid embolism c) Placental separation d) Aspiration

Amniotic fluid embolism Correct Explanation: With amniotic fluid embolism, symptoms may occur suddenly during or immediately after labor. The woman usually develops symptoms of acute respiratory distress, cyanosis, and hypotension.

After teaching a class about various methods for cervical ripening, the instructor determines that the teaching was successful when the class identifies which of the following as a surgical method? a) Prostaglandin b) Breast stimulation c) Amniotomy d) Laminaria

Amniotomy

A woman whose fetus in in the occiput-posterior position is experiencing increased back pain. Which is the best way for the nurse to help alleviate this back pain? a) Applying counter pressure to the back b) Applying a heating pad to the back c) Performing acupuncture on the back d) Applying ice to the back

Applying counter pressure to the back

The nurse identifies a nursing diagnosis of risk for injury related to possible effects of oxytocin therapy. Which of the following would the nurse do to ensure a positive outcome for the client? a) Assess contractions by using external monitor b) Start administering tocolytic therapy c) Turn down oxytocin administration by half d) Administer hydration and sedation frequently

Assess contractions by using external monitor

Which of the following postoperative interventions should a nurse perform when caring for a client who has undergone a cesarean birth? a) Ensure that the client does not cough or breathe deeply. b) Avoid early ambulation to prevent respiratory problems. c) Delay breastfeeding the newborn for a day. d) Assess uterine tone to determine fundal firmness.

Assess uterine tone to determine fundal firmness.

When educating the post-term pregnant patient, what should the nurse be sure to include to prevent fetal complications? a) Increase fluid intake to prevent dehydration. b) Be sure to monitor fetal movements daily. c) Monitor bowel movements. d) Be sure to measure 24-hour urine output daily.

Be sure to monitor fetal movements daily.

Amniotic fluid embolism is a true medical emergency. If it occurs during labor, what sign of fetal distress is usually noted? a) Arrested fetal movement b) Tachycardia c) Tachapnea d) Bradycardia

Bradycardia Explanation: In most cases, symptoms of amniotic fluid embolism occur suddenly during or immediately after labor. The woman usually develops symptoms of acute respiratory distress, cyanosis, and hypotension. If she is in labor, the fetus typically demonstrates signs of fetal distress, with bradycardia occurring in most cases.

The nurse is assessing the woman who has a forceps-assisted birth for complications. Which of the following would be least likely to occur in the mother? a) Infection of episiotomy b) Caput succedaneum c) Cervical lacerations d) Perineal hematoma

Caput succedaneum

The nurse is assessing the woman who has a forceps-assisted birth for complications. Which of the following would be least likely to occur in the mother? a) Perineal hematoma b) Infection of episiotomy c) Cervical lacerations d) Caput succedaneum

Caput succedaneum Correct Explanation: Caput succedaneum is a complication that may occur in the newborn of a woman who had a forceps- assisted birth. Maternal complications include tissue trauma, such as lacerations of the cervix, vagina, and perineum, hematoma, extension of episiotomy into the anus, hemorrhage, and infection.

A nurse is caring for obstetric clients. The nurse should be aware of which of the following as an indication for labor induction? a) Chorioamnionitis b) Abruptio placenta c) Complete placenta previa d) Transverse fetal lie

Chorioamnionitis

When assessing the following women, which would the nurse identify as being at the greatest risk for preterm labor? a. Woman who had twins in a previous pregnancy b. Client living in a large city c. Woman working full time as a computer programmer d. Client with a history of a previous preterm birth

Client with a history of a previous preterm birth

Which finding would lead the nurse to suspect that the fetus of a woman in labor is in a persistent occiput posterior position? a) Lack of cervical dilation past 2 cm b) Contractions most forceful in the middle of uterus rather than the fundus c) Fetal buttocks as the presenting part d) Complaints of severe back pain

Complaints of severe back pain

A client is giving birth when shoulder dystocia occurs in the fetus. The nurse recognizes that which of the following conditions in the client is likely to increase the risk for shoulder dystocia? a) Pendulous abdomen b) Diabetes c) Nullipara d) Preterm delivery

Diabetes

A woman's nurse-midwife tells her that the woman has developed dystocia. You would explain that this term means a) High blood pressure related to difficult labor. b) Potential for placental detachment. c) Difficult or abnormal labor. d) Muscle weakness related to prolonged labor.

Difficult or abnormal labor. Correct Explanation: Dystocia is a general term used to describe difficult or abnormal labor. Dystocia does not indicate high blood pressure related to difficult labor, a potential for placental detachment, nor muscle weakness related to prolonged labor.

You assess that the fetus of a woman is in an occiput posterior position. Which of the following identifies the way you would expect her labor to differ from others? a) Shorter dilatational stage of labor. b) Necessity for vacuum extraction for delivery. c) Need to have the baby manually rotated. d) Experience of additional back pain.

Experience of additional back pain.

The nurse is assisting a physician to attempt to manipulate the position of the fetus in utero from a breech to cephalic position. What does the nurse inform the patient the procedure is called? a) External version b) Vaginal manipulation c) Internal rotation d) External rotation

External version

When reviewing the medical record of a client, the nurse notes that the woman has a condition in which the fetus cannot physically pass through the maternal pelvis. The nurse interprets this as: a. Cervical insufficiency b. Contracted pelvis c. Maternal disproportion d. Fetopelvic disproportion

Fetopelvic disproportion

The nurse would anticipate a cesarean birth for a client who has which active infection present at the onset of labor? a. Hepatitis b. Herpes simplex virus c. Toxoplasmosis d. Human papillomavirus

Herpes simplex virus

At 31 weeks' gestation, a 37-year-old woman who has a history of preterm birth reports cramps, vaginal pain, and low, dull backache accompanied by vaginal discharge and bleeding. Her cervix is 2.1 cm long; she has fetal fibronectin in her cervical secretions, and her cervix is dilated 3 to 4 cm. For what do you prepare her? a) An emergency cesarean section b) Bed rest and hydration at home c) Hospitalization, tocolytic therapy, and IM corticosteroids d) Careful monitoring of fetal kick counts

Hospitalization, tocolytic therapy, and IM corticosteroids Explanation: At 31 weeks gestation, the goal would be to maintain the pregnancy as long as possible if the mother and fetus are tolerating continuation of the pregnancy. Stopping the contractions and placing the patient in the hospital allow for monitoring and a safe place if the woman continues and delivers. Administration of corticosteroids may help to develop the lungs and prepare for early preterm delivery. Sending the woman home is contraindicated in the scenario described. An emergency cesarean section is not indicated at this time. Monitoring fetal kick counts is typically done with a post-term pregnancy.

A client who was in active labor and whose cervix had dilated to 4 cm experiences a weakening in the intensity and frequency of her contractions and exhibits no further progress in labor. The nurse interprets this as a sign of: a. Hypertonic labor b. Precipitate labor c. Hypotonic labor d. Dysfunctional labor

Hypotonic labor

A nurse is assessing the cause of multiple gestations in clients. Which of the following factors should the nurse assess as contributors to increased probability of multiple gestations? a) Advanced maternal age b) Medications c) Adolescent pregnancies d) Infertility treatment

Infertility treatment

After a vaginal examination, the nurse determines that the client's fetus is in an occiput posterior position. The nurse would anticipate that the client will have: a. Intense back pain b. Frequent leg cramps c. Nausea and vomiting d. A precipitous birth

Intense back pain

A nurse is caring for a client who has been diagnosed with precipitous labor. For which of the following potential fetal complications should the nurse monitor as a result of precipitous labor? a) Cephalhematoma b) Facial nerve injury c) Facial lacerations d) Intracranial hemorrhage

Intracranial hemorrhage

At the hospital, a client is attached to the fetal monitor for uterine rupture. The nurse would assess for which pattern indicating change in the uterus impacting the fetus? a) Variable decelerations. b) Early decelerations. c) Late decelerations. d) Mild decelerations.

Late decelerations.

At the hospital, a client is attached to the fetal monitor for uterine rupture. The nurse would assess for which pattern indicating change in the uterus impacting the fetus? a) Early decelerations. b) Variable decelerations. c) Mild decelerations. d) Late decelerations.

Late decelerations. Explanation: When the fetus is being deprived of oxygen the fetus will demonstrated late decelerations on the fetal monitoring strip. This is an indication the mother is in need of further assessment. Early decelerations are a normal finding. Variable decelerations usually coincide with cord compression. (less)

Which action could the nurse initiate to reduce the discomfort of a woman in labor whose fetus is in an occiput posterior position?

Massage her lower back. Counterpressure against the woman's back by a support person can be helpful in reducing this type of pain.

Which of the following actions could you initiate to reduce the discomfort of a woman in labor whose fetus is in an occiput posterior position? a) Place her in a Trendelenburg position. b) Apply ice packs to her lower back. c) Massage her lower back. d) Urge her to maintain a prone position.

Massage her lower back. Correct Explanation: Counterpressure against the woman's back by a support person can be helpful in reducing this type of pain.

A laboring patient has been pushing without delivering the fetal shoulders. The physician determines the fetus is experiencing shoulder dystocia. What intervention can the nurse assist with to help with the delivery? a) Lamaze position b) Fundal pressure c) McRobert's maneuver d) Positioning the woman prone

McRobert's maneuver

You are assisting with delivery of the second child of a healthy young woman. Her pregnancy has been uneventful, and labor has been progressing well. The fetal head begins to deliver but instead of continuing to emerge, it retracts into the vagina. What should you try first? a) Apply pressure to the fundus. b) Attempt to push one of the fetus' shoulders in a clockwise or counterclockwise motion. c) Zavanelli's maneuver d) McRobert's maneuver

McRobert's maneuver

A laboring client has been pushing without delivering the fetal shoulders. The primary care provider determines the fetus is experiencing shoulder dystocia. What intervention can the nurse assist with to help with the birth?

McRobert's maneuver The McRobert's maneuver is frequently successful and often tried first. It requires assistance from two people. Two nurses place the client in the lithotomy position, while each holds a leg and sharply flexes the leg toward the woman's shoulders. This opens the pelvis to its widest diameters and allows the anterior shoulder to deliver in almost half of the cases.

A laboring patient has been pushing without delivering the fetal shoulders. The physician determines the fetus is experiencing shoulder dystocia. What intervention can the nurse assist with to help with the delivery? a) Positioning the woman prone b) Fundal pressure c) Lamaze position d) McRobert's maneuver

McRobert's maneuver Correct Explanation: The McRobert's maneuver is frequently successful and often tried first. It requires assistance from two people. Two nurses place the patient in the lithotomy position, while each holds a leg and sharply flexes the leg toward the woman's shoulders. This opens the pelvis to its widest diameters and allows the anterior shoulder to deliver in almost half of the cases.

Shoulder dystocia is a true medical emergency that can cause fetal demise because the baby cannot be delivered. Stuck in the birth canal, the infant cannot take its first breath. What is the first maneuver tried to deliver an infant with shoulder dystocia? a) McGeorge maneuver b) McRonald Maneuver c) McRoberts maneuver d) McDonald maneuver

McRoberts maneuver McRoberts maneuver is an intervention that is frequently successful in cases of shoulder dystocia, and is often tried first. McRoberts requires the assistance of two individuals. Two nurses are ideal; however, a support person or a technician can serve as the second assistant. With the woman in lithotomy position, each nurse holds one leg and sharply flexes the leg toward the woman's shoulders. This opens the pelvis to its widest diameters and allows the anterior shoulder to deliver in almost half of the cases.

Shoulder dystocia is a true medical emergency that can cause fetal demise because the baby cannot be born. Stuck in the birth canal, the infant cannot take its first breath. What is the first maneuver tried to deliver an infant with shoulder dystocia?

McRoberts maneuver McRoberts maneuver is an intervention that is frequently successful in cases of shoulder dystocia, and it is often tried first. McRoberts requires the assistance of two individuals. Two nurses are ideal; however, a support person or a technician can serve as the second assistant. With the woman in lithotomy position, each nurse holds one leg and sharply flexes the leg toward the woman's shoulders. This opens the pelvis to its widest diameters and allows the anterior shoulder to deliver in almost half of the cases.

A client is experiencing shoulder dystocia during delivery. Which of the following should the nurse identify as risks to the fetus in such a condition? a) Extensive lacerations b) Infection c) Nerve damage d) Bladder injury

Nerve damage

A client with full-term pregnancy who is not in active labor has been ordered oxytocin intravenously. Which of the following is a contraindication for oxytocin administration? a) Dysfunctional labor pattern b) Postterm status c) Prolonged ruptured membranes d) Overdistended uterus

Overdistended uterus

A patient is 23 weeks gestation and was admitted for induction and delivery after noting the infant was an intrauterine fetal death. The patient had fallen 3 days prior to the diagnosis and landed on her side. What is the most likely attributable cause to the fetal death? a) Genetic abnormality b) Preeclampsia c) Placental abruption d) Premature rupture of membranes

Placental abruption

A patient is 23 weeks gestation and was admitted for induction and delivery after noting the infant was an intrauterine fetal death. The patient had fallen 3 days prior to the diagnosis and landed on her side. What is the most likely attributable cause to the fetal death? a) Placental abruption b) Preeclampsia c) Premature rupture of membranes d) Genetic abnormality

Placental abruption Correct Explanation: The most common cause of fetal death after a trauma is placental abruption, where the placenta separates from the uterus and the fetus is not able to survive. Genetic abnormalities typically cause spontaneous abortion in the first trimester. The scenario does not indicate that there has been a premature rupture of membranes, nor the possibility of preeclamsia.

When caring for a client requiring a forceps-assisted birth, the nurse would be alert for which of the following? a) Increased risk for cord entanglement b) Increased risk for uterine rupture c) Damage to the maternal tissues d) Potential lacerations and bleeding

Potential lacerations and bleeding

When caring for a client requiring a forceps-assisted birth, the nurse would be alert for which of the following? a) Potential lacerations and bleeding b) Increased risk for uterine rupture c) Damage to the maternal tissues d) Increased risk for cord entanglement

Potential lacerations and bleeding Correct Explanation: Forcible rotation of the forceps can cause potential lacerations and bleeding.. Cervical ripening increases the risk for uterine rupture in a client attempting vaginal birth after undergoing at least one previous cesarean birth. There is an increased risk for cord entanglement in multiple pregnancies. Damage to the maternal tissues happens if the cup slips off the fetal head and the suction is not released.

A client is admitted to the health care facility. The fetus has a gestational age of 42 weeks and is suspected to have cephalopelvic disproportion. Which should the nurse do next?

Prepare the client for a cesarean birth. Cephalopelvic disproportion is associated with postterm pregnancy. This client will not be able to vaginally give birth and should be prepared for a cesarean birth. Lithotomy position, artificial rupture of membranes, and oxytocin are interventions for a vaginal birth.

A multipara presents to the hospital after 2 hours of labor. The fetus is presenting in transverse lie. You notify the physician and take what action? a) Apply pressure to the woman's lower back with a fisted hand. b) Include a set of piper forceps when you prep the table. c) Prepare to assist with external version or prep for a cesarean section delivery. d) Assist with Nitrazine and fern tests.

Prepare to assist with external version or prep for a cesarean section delivery.

A multipara presents to the hospital after 2 hours of labor. The fetus is presenting in transverse lie. You notify the physician and take what action? a) Apply pressure to the woman's lower back with a fisted hand. b) Include a set of piper forceps when you prep the table. c) Assist with Nitrazine and fern tests. d) Prepare to assist with external version or prep for a cesarean section delivery.

Prepare to assist with external version or prep for a cesarean section delivery. Correct Explanation: Transverse lie is a fetal malposition and is a cause for labor dystocia. The fetus would need to be turned to the occipital position or be delivered via a cesarean delivery. Piper forceps are used in the delivery of a fetus that is in the breech position. Nitrazine and fern tests are done to assess if amniotic fluid is leaking from the sac into the vagina. Counter pressure applied to the lower back with a fisted hand sometimes helps the woman to cope with the "back labor" that is characteristic of occiput posterior positioning

The second-year nursing student taking an obstetrics course correctly attributes which descriptions to the term dystocia? Select all that apply.

Progress of labor deviates from normal. Labor is slow. Dystocia is said to exist when the progress of labor deviates from normal and is slow.

Which action would be most appropriate for the woman who experiences dysfunctional labor in the first stage of labor?

Provide ongoing communication about what is happening. Dysfunctional labor at any point is frustrating to women. Maintaining open lines of communication at least keeps the woman well informed about what is happening.

Which intervention would be most appropriate for the woman experiencing dystocia related to problems involving the psyche? a) Preparing the woman for an amniotomy b) Administering oxytocin c) Encouraging the woman to assume a hands-and-knees position d) Providing a comfortable environment with dim lighting

Providing a comfortable environment with dim lighting Explanation: Comfort measures minimize the woman's stress and promote relaxation so that she can work more effectively with the forces of labor. Oxytocin would be appropriate for the woman experiencing hypotonic uterine dysfunction (problem with the powers). An amniotomy may be used with hypertonic uterine dysfunction to augment labor. A hands-and-knees position helps to promote fetal head rotation with a persistent occiput posterior position.

Which action by the nurse would be least effective in assisting a couple who have experienced intrauterine fetal demise? a) Give the parents a lock of the infant's hair b) Assist the family in making arrangements for their stillborn infant c) Allow the couple to spend as much time as they want with their stillborn infant. d) Refrain from discussing the situation with the couple

Refrain from discussing the situation with the couple

A multigravida presents at 31 weeks' gestation with signs and symptoms of preterm labor. The diagnosis is confirmed and she is admitted and given magnesium sulfate. What must you report as part of her care? a) Respiratory depression, hypotension, absent tendon reflexes b) Pain in the abdomen, shoulder, or back c) Severe lower back pain, leg cramps, sweating d) Low potassium or elevated glucose, tachycardia, chest pain

Respiratory depression, hypotension, absent tendon reflexes

Labor dystocia is an abnormally progression of labor. It is the most common cause of primary caesarian delivery. When is it most common for labor dystocia to occur? a) Second stage of labor b) Fourth stage of labor c) Third stage of labor d) First stage of labor

Second stage of labor

Labor dystocia is an abnormally progression of labor. It is the most common cause of primary caesarian delivery. When is it most common for labor dystocia to occur? a) Fourth stage of labor b) Third stage of labor c) First stage of labor d) Second stage of labor

Second stage of labor Correct Explanation: Labor dystocia can occur in any stage of labor, although it occurs most commonly once the woman is in active labor or when she reaches the second stage of labor.

The nurse would prepare a client for amnioinfusion when which of the following occurs? a) Severe variable decelerations are due to cord compression b) Maternal pushing is compromised due to anesthesia c) Fetal presenting part fails to rotate fully and descend in the pelvis d) The fetus shows non-reassuring fetal heart rate patterns

Severe variable decelerations are due to cord compression

A woman receiving an oxytocin infusion for labor induction develops contractions that occur every minute and last 75 seconds. Uterine resting tone remains at 20 mm Hg. Which action would be most appropriate? a) Slow the oxytocin infusion to the initial rate. b) Stop the infusion immediately. c) Continue to monitor contractions and fetal heart rate. d) Notify the birth attendant.

Stop the infusion immediately. Explanation: The woman is exhibiting signs of uterine hyperstimulation, which necessitate stopping the oxytocin infusion immediately to prevent further complications. Once the infusion is stopped, the nurse should notify the birth attendant and continue to monitor the woman's contractions and fetal heart rate.

A pregnant woman at her first office visit tells the nurse, "I hope I have twins. It seems like all my friends have delivered twins lately!" What should the nurse tell the patient about multiple births at this time? a) The incidence is decreasing. b) Incidence of twins is consistently approximately 1 in 100 conceptions. c) People giving birth at younger ages is resulting in more twins. d) The incidence is increasing.

The incidence is increasing.

Which of the following would be appropriate for the use of low forceps? a) The fetal scalp is visible at the introitus without spreading the labia b) The leading point of fetal skull is at or above station +2, not on the pelvic floor c) The fetal skull has reached the pelvic floor, with the fetal head at the perineum d) The fetal head is engaged but the leading point of the skull is less than +2

The leading point of fetal skull is at or above station +2, not on the pelvic floor

Which statement describes why hypertonic contractions tend to become very painful?

The myometrium becomes sensitive from the lack of relaxation and anoxia of uterine cells. Hypertonic contractions cause uterine cell anoxia, which is painful.

A client with a pendulous abdomen and uterine fibroid tumors had just begun labor and arrived at the hospital. After examining the client, the physician informs the nurse that the fetus appears to be malpositioned in the uterus. Which fetal position or presentation should the nurse most expect in this woman? a) Anterior fetal position b) Occipitoposterior position c) Cephalic presentation d) Transverse lie

Transverse lie

In vasa previa, the umbilical vessels of a velamentous cord insertion cross the cervical os and therefore deliver before the fetus.

True

In vasa previa, the umbilical vessels of a velamentous cord insertion cross the cervical os and therefore deliver before the fetus. a) False b) True

True

In vasa previa, the umbilical vessels of a velamentous cord insertion cross the cervical os and therefore deliver before the fetus. a) True b) False

True

Hypertonic labor is labor that is characterized by short, irregular contractions without complete relaxation of the uterine wall in between contractions. Hypertonic labor can be caused by an increased sensitivity to oxytocin. What would you do for a patient who is in hypertonic labor because of oxytocin augmentation? a) Increase the methotrexate b) Increase the pitocin c) Turn off the pitocin d) Turn off the methotrexate

Turn off the pitocin Correct Explanation: Hypertonic labor may result from an increased sensitivity of uterine muscle to oxytocin induction or augmentation. Treatment for this iatrogenic cause of hypertonic labor is to decrease or shut off the oxytocin infusion.

A nurse is assessing a full-term patient in labor and determines the fetus is occiput posterior. The patient states that all her discomfort is in her lower back. What intervention can the nurse provide that will help alleviate this discomfort? a) Place the patient supine with the head of bed elevated 30 degrees. b) Apply a warm washcloth to the lower back. c) Have the physician administer a pudendal block. d) Use a fist to apply counter pressure to the lower back.

Use a fist to apply counter pressure to the lower back.

A nurse is assessing a full-term client in labor and determines the fetus is occiput posterior. The client states that all her discomfort is in her lower back. What intervention can the nurse provide that will help alleviate this discomfort?

Use a fist to apply counter pressure to the lower back. Counter pressure applied to the lower back with a fisted hand sometimes helps the woman to cope with the "back labor" characteristic of the occiput posterior position.

A client in the first stage of labor is diagnosed with dystocia involving the powers of labor. Which of the following would the nurse identify as the problem? a) Pelvis is either android type or platypelloid type b) Contractions are insufficient to cause fetus descent c) Uterine contractions are too weak or uncoordinated d) Fetus is in a different position or presentation

Uterine contractions are too weak or uncoordinated

While in labor a woman with a prior history of cesarean birth complains of light-headedness and dizziness. The nurse assesses the patient and notes an increase in pulse and decrease in blood pressure from the vital signs 15 minutes prior. What might the nurse consider as a possible cause for the symptoms? a) Uterine rupture b) Placentea previa c) Umbilical cord compression d) Hypertonic uterus

Uterine rupture Correct Explanation: The patient with any prior history of uterus surgery is at increased risk for a uterine rupture. A falling blood pressure and increasing pulse is a sign of hemorrhage and in this patient a uterine rupture needs to be a first consideration. The scenario does not indicate a hypertonic uterus, a placenta previa, nor umbilical cord compression.

When administering oxytocin to a woman in labor, the nurse would be alert for which of the following? a) Hypertension b) Uterine hypotonicity c) Fetal distress d) Water intoxication

Water intoxication Explanation: Oxytocin can lead to water intoxication and can cause hypotension. Uterine hypertonicity is a possible adverse effect of oxytocin administration. Oxytocin does not cross the placental barrier, and no fetal problems have been observed.

The nurse is caring for a client experiencing a prolonged second stage of labor. The nurse would place priority on preparing the client for which intervention?

a forceps and vacuum-assisted birth A forceps-and-vacuum-assisted birth is required for the client having a prolonged second stage of labor. The client may require a cesarean birth if the fetus cannot be delivered with assistance. A precipitous birth occurs when the entire labor and birth process occurs very quickly. Artificial rupture of membranes is done during the first stage of labor.

A woman experiences an amniotic fluid embolism as the placenta is delivered. Your first action would be to a) increase her intravenous fluid infusion rate. b) tell the woman to take short, catchy breaths. c) administer oxygen by mask. d) put firm pressure on the fundus of her uterus.

administer oxygen by mask.

A woman experiences an amniotic fluid embolism as the placenta is delivered. The nurse's first action would be to:

administer oxygen by mask. An amniotic embolism quickly becomes a pulmonary embolism. The woman needs oxygen to compensate for the sudden blockage of blood flow through her lungs.

A woman experiences an amniotic fluid embolism as the placenta is delivered. Your first action would be to a) administer oxygen by mask. b) increase her intravenous fluid infusion rate. c) put firm pressure on the fundus of her uterus. d) tell the woman to take short, catchy breaths.

administer oxygen by mask. Correct Explanation: An amniotic embolism quickly becomes a pulmonary embolism. The woman needs oxygen to compensate for the sudden blockage of blood flow through her lungs.

A nurse assesses a client in labor and suspects hypotonic uterine dysfunction. Which intervention would the nurse expect to include in the plan of care for this client?

administering oxytocin Oxytocin would be appropriate for the woman experiencing hypotonic uterine dysfunction (problem with the powers). Comfort measures minimize the woman's stress and promote relaxation so that she can work more effectively with the forces of labor. An amniotomy may be used with hypertonic uterine dysfunction to augment labor. A hands-and-knees position helps to promote fetal head rotation with a persistent occiput posterior position.

A woman whose fetus in in the occiput-posterior position is experiencing increased back pain. Which is the best way for the nurse to help alleviate this back pain?

applying counter pressure to the back Counter pressure applied to the lower back with a fisted hand sometimes helps the woman cope with "back labor" associated with occiput-posterior positioning. The others are not recommended or used techniques for a woman in labor with back pain.

After an hour of oxytocin therapy, a woman in labor states she feels dizzy and nauseated. The nurse's best action would be to:

assess the rate of flow of the oxytocin infusion. A toxic effect of oxytocin therapy is water intoxication. Symptoms include dizziness and nausea. Assessing and slowing the infusion rate will relieve symptoms.

After an hour of oxytocin therapy, a woman in labor states she feels dizzy and nauseated. Your best action would be to a) administer oral orange juice for added potassium. b) assess her vaginally for full dilation. c) assess the rate of flow of the oxytocin infusion. d) instruct her to breathe in and out rapidly.

assess the rate of flow of the oxytocin infusion. Correct Explanation: A toxic effect of oxytocin therapy is water intoxication. Symptoms include dizziness and nausea. Assessing and slowing the infusion rate will relieve symptoms.

A client is experiencing shoulder dystocia during birth. The nurse would place priority on performing which assessment postbirth?

brachial plexus assessment The nurse should identify nerve damage as a risk to the fetus in cases of shoulder dystocia. Other fetal risks include asphyxia, clavicle fracture, central nervous system injury or dysfunction, and death. Extensive lacerations is a poor maternal outcome due to the occurrence of shoulder dystocia. Cleft palate and cardiac anomalies are not related to shoulder dystocia.

Which finding would lead the nurse to suspect that the fetus of a woman in labor is in hypertonic uterine dysfuction?

contractions most forceful in the middle of uterus rather than the fundus Contractions that are more forceful in the midsection of the uterus rather than in the fundus suggest hypertonic uterine dysfunction. Reports of severe back pain are associated with a persistent occiput posterior position due to the pressure of the fetal head on the woman's sacrum and coccyx. Cervical dilation that has not progressed past 2 cm is associated with dysfunctional labor. A breech position is one in which the fetal presenting part is the buttocks or feet.

A woman is admitted to the labor suite with contractions every five minutes lasting one minute. She is postterm and has oligohydramnios. What does this increase the risk of during birth?

cord compression Oligohydramnios and meconium staining of the amniotic fluid are common complications of postterm pregnancy. Oligohydramnios increases the incidence of cord compression, which can lead to fetal distress during labor.

The nurse providing care for a woman with preterm labor on magnesium sulfate would include which assessment for safe administration of the drug?

deep tendon reflexes (DTR)s Assessing deep tendon reflexes hourly in a client receiving magnesium sulfate is appropriate as depressed DTRs are a sign of magnesium toxicity. Elevated blood glucose is a fetal side effect but not noted to assess with the mother. Assessing for depressed respiration and hypotension not tachypnea or tachycardia would be appropriate assessments needed for the safe administration of magnesium sulfate.

After an hour of administering oxytocin intravenously, you assess a woman's contractions to be 80 seconds in length. Your first action would be to a) continue to monitor contraction duration every 2 hours. b) increase the flow rate of the main line infusion. c) discontinue the oxytocin infusion. d) slow the infusion to under 10 gtt per minute.

discontinue the oxytocin infusion.

The nursing student doing a clinical rotation in labor and delivery has noticed numerous women expressing various emotions during labor. The student's preceptor informs the student that these emotions can lead to psychological stress, which in turn can cause which of the following complications? a) pulmonary emboli b) dystocia c) deep vein thrombosis d) premature labor

dystocia Correct Explanation: Many women experience an array of emotions during labor, which may include fear, anxiety, helplessness, desire to be alone, and weariness. These emotions can lead to psychological stress, which indirectly can cause dystocia.

The nurse assesses that the fetus of a woman is in an occiput posterior position. Which description identifies the way the nurse would expect the client's labor to differ from others?

experience of additional back pain Most women whose fetus is in a posterior position experience back pain while in labor. Pressure against the back by a support person often reduces this type of pain. An occiput posterior position does not make for a shorter dilatational stage of labor, it does not indicate the need to have the baby manually rotated, and it does not indicate a necessity for a vacuum extraction birth.

The nurse is assisting a primary care provider to attempt to manipulate the position of the fetus in utero from a breech to cephalic position. What does the nurse inform the client the procedure is called?

external version External version is the process of manipulating the position of the fetus in order to try to turn the fetus to a cephalic presentation.

The client is 35 weeks of gestation and is being admitted for vaginal bleeding. She is stable at the time of admission. The priority nursing assessment for the client is for:

fetal heart tones. When a client is admitted for vaginal bleeding and is stable, the next priority assessment is to determine if the fetus is viable. The other options are not a higher priority than fetal heart tones.

At 31 weeks' gestation, a 37-year-old woman who has a history of preterm birth reports cramps, vaginal pain, and low, dull backache accompanied by vaginal discharge and bleeding. Her cervix is 2.1 cm long; she has fetal fibronectin in her cervical secretions, and her cervix is dilated 3 to 4 cm. For what does the nurse prepare her?

hospitalization, tocolytic therapy, and IM corticosteroids At 31 weeks gestation, the goal would be to maintain the pregnancy as long as possible if the mother and fetus are tolerating continuation of the pregnancy. Stopping the contractions and placing the client in the hospital allow for monitoring and a safe place if the woman continues and gives birth. Administration of corticosteroids may help to develop the lungs and prepare for early preterm birth. Sending the woman home is contraindicated in the scenario described. An emergency cesarean birth is not indicated at this time. Monitoring fetal kick counts is typically done with a postterm pregnancy.

A nursing student has learned that precipitous labor is when the uterus contracts so frequently and with such intensity that a very rapid birth will take place. This means the labor will be completed in which span of time?

less than 3 hours Precipitous labor is completed in less than 3 hours.

A nursing student correctly identifies the most desirable position to promote an easy delivery as which of the following? a) occiput anterior b) breech c) shoulder dystocia d) face and brow

occiput anterior

A nursing student correctly identifies the most desirable position to promote an easy birth as which position?

occiput anterior Any presentation other than occiput anterior or a slight variation of the fetal position or size increases the probability of dystocia.

A gravida 7, para 6 woman is in the hospital only 15 minutes when she begins to deliver precipitously. The fetal head begins to deliver as you walk into the labor room. Your best action would be to a) assess blood pressure and pulse to detect placental bleeding. b) attach a fetal monitor to determine fetal status. c) ask her to push with the next contraction so delivery is rapid. d) place a hand gently on the fetal head to guide delivery.

place a hand gently on the fetal head to guide delivery.

A gravida 7, para 6 woman is in the hospital only 15 minutes when she begins to deliver precipitously. The fetal head begins to deliver as you walk into the labor room. Your best action would be to a) attach a fetal monitor to determine fetal status. b) ask her to push with the next contraction so delivery is rapid. c) place a hand gently on the fetal head to guide delivery. d) assess blood pressure and pulse to detect placental bleeding.

place a hand gently on the fetal head to guide delivery. Correct Explanation: If a head is controlled as it delivers, trauma to internal vessels or to the maternal cervix is less apt to occur.

A client is at 23 weeks' gestation and was admitted for induction and birth after noting the infant was an intrauterine fetal death. The client had fallen 3 days prior to the diagnosis and landed on her side. What is the most likely attributable cause to the fetal death?

placental abruption The most common cause of fetal death after a trauma is placental abruption, where the placenta separates from the uterus, and the fetus is not able to survive. Genetic abnormalities typically cause spontaneous abortion in the first trimester. The scenario does not indicate that there has been a premature rupture of membranes or the possibility of preeclamsia.

The nursing student doing a clinical obstetrics rotation correctly picks which of the following to label a pregnancy that continues past the end of the 42nd week of gestation? a) term pregnancy b) post-term pregnancy c) preterm pregnancy d) none of the above

post-term pregnancy Correct Explanation: A term pregnancy usually lasts 38 to 42 weeks. A post-term pregnancy continues past the end of the 42nd week of gestation. A preterm pregnancy ends before the 34th week of gestation.

When caring for a client requiring a forceps-assisted birth, the nurse would be alert for:

potential lacerations and bleeding. Forcible rotation of the forceps can cause potential lacerations and bleeding. Cervical ripening increases the risk for uterine rupture in a client attempting vaginal birth after undergoing at least one previous cesarean birth. There is an increased risk for cord entanglement in multiple pregnancies. Damage to the maternal tissues happens if the cup slips off the fetal head and the suction is not released.

A patient is told that she is already completely effaced and 9 cm dilated, and that the fetal head is showing. Contrary to the nurse's instructions, the patient begins to push. Before the physician can enter the room, the woman delivers the baby with only the nurse in attendance. This is an example of which of the following? a) protraction disorder b) labor dystocia c) precipitous delivery d) uterine dysfunction

precipitous delivery

A woman having contractions comes to the emergency department. She tells the nurse that she is at 34 weeks' gestation. The nurse examines her and finds that she is already effaced and dilated 2 cm. What is this woman demonstrating? a) macrosomia b) dystocia c) normal labor d) preterm labor

preterm labor

A nursing instructor identifies which of the following as increasing the chances of infection when coupled with prolonged labor? a) multiple births b) number of previous pregnancies c) age of mother d) ruptured membranes

ruptured membranes Correct Explanation: The risk for infection increases during prolonged labor particularly in association with ruptured membranes. The other options do not increase the risk of infection during labor.

A woman the nurse is caring for during labor is having contractions 2 minutes apart but rarely over 50 mm Hg in strength; the resting tone is high, 20 to 25 mm Hg. She asks what she can do to make contractions more effective. The nurse's best response would be that:

she needs to rest because her contractions are hypertonic. These contractions appear to be hypertonic because of the high resting tone. Hypertonic contractions occur because the uterus is being overstimulated or erratically stimulated. Rest is effective in helping contractions become more productive.

A nurse is assessing the following antenatal clients. Which client is at highest risk for having a multiple gestation?

the 41-year-old client who conceived by in vitro fertilization The nurse should assess infertility treatment as a contributor to increased probability of multiple gestations. Multiple gestations do not occur with an adolescent birth; instead, chances of multiple gestations are known to increase due to the increasing number of women giving birth at older ages.

A client with a pendulous abdomen and uterine fibroid tumors had just begun labor and arrived at the hospital. After examining the client, the primary care provider informs the nurse that the fetus appears to be malpositioned in the uterus. Which fetal position or presentation should the nurse most expect in this woman?

transverse lie A transverse lie, in which the fetus is more horizontal than vertical, occurs in women with pendulous abdomens, with uterine fibroid tumors that obstruct the lower uterine segment, with contraction of the pelvic brim, with congenital abnormalities of the uterus, or with hydramnios. Anterior fetal position and cephalic presentation are normal conditions. Occipitoposterior position tends to occur in women with android, anthropoid, or contracted pelves.

A nurse preceptor asks a student to list commonly used diagnostic tests for preterm labor risk assessment. Which of the following tests should the student include? (Select all that apply.) a) thyroid level b) amniotic fluid analysis c) U/A d) CBC e) arterial blood gases

• CBC • U/A • amniotic fluid analysis

A client is 2 weeks past her due date, and her physician is considering whether to induce labor. Which of the following conditions must be present before induction can take place? (Select all that apply.) a) The fetus is in a longitudinal lie b) The cervix is ripe c) A presenting part is engaged d) Absence of eclampsia e) Cephalopelvic disproportion is present f) Normal maternal blood pressure

• The fetus is in a longitudinal lie • The cervix is ripe • A presenting part is engaged

The nurse who works at the local health department is preparing to give a talk on post-term pregnancies. She wants to include the fetal risks. Which of the following should she include? (Check all that apply.) a) cephalopelvic disproportion b) brachial plexus injuries c) shoulder dystocia d) macrosomia e) failure to thrive

• cephalopelvic disproportion • brachial plexus injuries • shoulder dystocia • macrosomia Correct Explanation: Fetal risks associated with a post-term pregnancy include macrosomia, shoulder dystocia, brachial plexus injuries, and cephalopelvic disproportion. Failure to thrive is more frequently associated with newborns who are of a low birth weight.

The nurse who works at the local health department is preparing to give a talk on post-term pregnancies. She wants to include the fetal risks. Which of the following should she include? (Check all that apply.) a) brachial plexus injuries b) failure to thrive c) macrosomia d) cephalopelvic disproportion e) shoulder dystocia

• macrosomia • shoulder dystocia • brachial plexus injuries • cephalopelvic disproportion

A patient who has been in prolonged labor reports extreme back pain. She asks why her back hurts so much. The best response by the nurse would be which of the following? a) "Let me help you out of bed to try walking it off." b) "Different fetal positions can cause prolonged labor and back pain." c) "This is just a normal part of labor." d) "Perhaps you have been in one position for too long."

"Different fetal positions can cause prolonged labor and back pain." Correct Explanation: Fetal malposition can cause prolonged labor. A labor complicated by occiput posterior position is usually prolonged and characterized by maternal perception of increased intensity of back discomfort. The other answers do not address the patient's question.

The nursing student demonstrates an understanding of dystocia with which of the following statements? a) "Dystocia cannot be diagnosed until just before delivery." b) "Dystocia is diagnosed at the start of labor." c) "Dystocia is diagnosed after labor has progressed for a time." d) "Dystocia is not diagnosed until after the delivery."

"Dystocia is diagnosed after labor has progressed for a time."

The nursing student demonstrates an understanding of dystocia with which of the following statements? a) "Dystocia is not diagnosed until after the delivery." b) "Dystocia is diagnosed at the start of labor." c) "Dystocia is diagnosed after labor has progressed for a time." d) "Dystocia cannot be diagnosed until just before delivery."

"Dystocia is diagnosed after labor has progressed for a time." Explanation: Nursing management of the woman with dystocia, regardless of etiology, requires patience. The nurse needs to provide physical and emotional support to the patient and family. Dystocia is diagnosed not at the start of labor, but rather after it has progressed for a time.

A pregnant patient at 32 weeks' gestation calls the clinic and informs the nurse that she thinks her membranes are leaking. She states that some clear fluid has run down her leg. What is the best response by the nurse? a) "It is best for you to visit a hospital immediately. They can use a nitrazine strip to determine if it is amniotic fluid." b) "You may have just passed some urine. If it were amniotic fluid, there would be much more than that." c) "There's nothing to worry about if you passed only a little bit. The membranes will seal back over." d) "Go to the hospital now, because this could be very dangerous for the baby."

"It is best for you to visit a hospital immediately. They can use a nitrazine strip to determine if it is amniotic fluid."

A pregnant patient at 32 weeks' gestation calls the clinic and informs the nurse that she thinks her membranes are leaking. She states that some clear fluid has run down her leg. What is the best response by the nurse? a) "There's nothing to worry about if you passed only a little bit. The membranes will seal back over." b) "It is best for you to visit a hospital immediately. They can use a nitrazine strip to determine if it is amniotic fluid." c) "You may have just passed some urine. If it were amniotic fluid, there would be much more than that." d) "Go to the hospital now, because this could be very dangerous for the baby."

"It is best for you to visit a hospital immediately. They can use a nitrazine strip to determine if it is amniotic fluid." Correct Explanation: The practitioner will perform a speculum examination, looking for pooling of amniotic fluid, and then test the fluid with nitrazine paper, which turns blue in the presence of amniotic fluid. Preterm premature rupture of membranes occurs when the rupture of the amniotic sac before the onset of labor happens in a woman who is less than 37 weeks' gestation.

A patient is 32 weeks gestation and sent home on modified bedrest for preterm labor. She is on tocolyitics and wants to know when she can have intercourse again with her husband. What is the most appropriate response by the nurse? a) "You will not be able to have intercourse again until 6 weeks after you deliver." b) "The need to keep the infant safe should be of more concern than when to have sex." c) "That is a question to ask your health care provider, at this point you are on pelvic rest to try and stop any further labor." d) "Intercourse has nothing to do with preterm labor; you can have sex with your husband."

"That is a question to ask your health care provider, at this point you are on pelvic rest to try and stop any further labor." Explanation: The patient needs to be on pelvic rest until the health care provider says otherwise. The intercourse can cause excitability in the uterus and encourage cervical softening and should be avoided unless the provider gives the OK. Option A is incorrect as it may be giving misinformation to the patient. Option B does not answer the patient's question so it is incorrect. Option D also gives misinformation to the patient and is incorrect.

During labor, a woman at 41 weeks gestation notes her amniotic fluid is leaking and is green in color. She is asking the nurse why the fluid is green. What is an appropriate response by the nurse? a) "This is meconium-stained fluid from the baby." b) "You have an infection and need antibiotics." c) "Green might be a yeast infection and we need to culture the discharge." d) "Amniotic fluid is normally green."

"This is meconium-stained fluid from the baby." Correct Explanation: Green tinted amniotic fluid is most often a sign of the infant having a bowel movement in the uterus, called mecnomium-stained fluid. This is more typical in a post-dates pregnancy. Green-stained amniotic fluid is not a normal color for amniotic fluid. However, it does not mean the mother has an infection and needs antibiotics, nor does it does mean there might be a yeast infection present or indicate the need for a culture of the fluid.

A nursing student is learning about fetal presentation. The nursing instructor realizes a need for further instruction when the student makes which of the following statements?

"Transverse lie is the same as when the fetal buttocks present to the birth canal." In most term pregnancies the fetus presents head down. In a breech presentation, the fetal buttocks, feet, or both present to the birth canal. Transverse lie is the same as shoulder presentation.

The experienced labor and delivery nurse knows to evaluate progress in active labor by using which simple rule? a) 1/2 cm/hour for cervical dilation b) 1 cm/hour for cervical dilation c) 2 cm/hour for cervical dilation d) 1/4 cm/hour for cervical dilation

1 cm/hour for cervical dilation Correct Explanation: In evaluating the progress in active labor, the nurse uses the simple rule of 1 cm/hour for cervical dilation.

A woman is to undergo labor induction. The nurse determines that the woman requires cervical ripening if her Bishop score is:

5. A Bishop score less than 6 usually indicates that a cervical ripening method should be used before labor induction.

A nursing student has learned that precipitous labor is when the uterus contracts so frequently and with such intensity that a very rapid birth will take place. This means the labor will be completed in which span of time? a) < 3 hours b) < 5 hours c) < 8 hours d) < 4 hours

< 3 hours

A nursing student has learned that precipitous labor is when the uterus contracts so frequently and with such intensity that a very rapid birth will take place. This means the labor will be completed in which span of time? a) < 8 hours b) < 4 hours c) < 3 hours d) < 5 hours

< 3 hours Correct Explanation: Precipitous labor is completed in less than 3 hours.

Why is it important for the nurse to thoroughly assess maternal bladder and bowel status during labor? a) A full bladder or rectum can impede fetal descent. b) If the woman has a full bladder, labor may be uncomfortable for her. c) A full rectum can cause diarrhea. d) If the woman's bladder is distended, it may rupture.

A full bladder or rectum can impede fetal descent. Explanation: Throughout labor the nurse needs to assess the woman's fluid balance status as well as check skin turgor and mucous membranes. In addition she needs to monitor the bladder and bowel status. A full bladder or rectum can impede fetal descent.

A woman is going to have labor induced with oxytocin. Which statement reflects the induction technique the nurse anticipates the primary care provider will prescribe?

Administer oxytocin diluted as a "piggyback" infusion. Oxytocin is always infused in a secondary or "piggyback" infusion system so it can be halted quickly if overstimulation of the uterus occurs.

A woman is going to have labor induced with oxytocin. Which statement below reflects the induction technique you anticipate her primary-care provider will order? a) Administer oxytocin diluted in the main intravenous fluid. b) Administer Pitocin in two divided intramuscular sites. c) Administer Pitocin in a 20 cc bolus of saline. d) Administer oxytocin diluted as a "piggyback" infusion.

Administer oxytocin diluted as a "piggyback" infusion. Correct Explanation: Pitocin is always infused in a secondary or "piggyback" infusion system so it can be halted quickly if overstimulation of the uterus occurs.

After teaching a class about various methods for cervical ripening, the instructor determines that the teaching was successful when the class identifies which of the following as a surgical method? a) Laminaria b) Amniotomy c) Breast stimulation d) Prostaglandin

Amniotomy Correct Explanation: Amniotomy is considered a surgical method of cervical ripening. Breast stimulation is considered a nonpharmacologic method for ripening the cervix. Laminaria is a hygroscopic dilator that mechanically causes cervical ripening. Prostaglandins are pharmacologic methods for cervical ripening.

The nurse identifies a nursing diagnosis of risk for injury related to possible effects of oxytocin therapy. Which of the following would the nurse do to ensure a positive outcome for the client? a) Administer hydration and sedation frequently b) Turn down oxytocin administration by half c) Assess contractions by using external monitor d) Start administering tocolytic therapy

Assess contractions by using external monitor Correct Explanation: In a client with the risk for injury, continuous assessment of contractions using external monitor and palpation to ensure the presence of a low resting tone will assist in collecting information about labor and the need for further intervention. Turning down oxytocin administration by half is required if hyperstimulation occurs, not to prevent it. Tocolytic therapy is generally employed when preterm labor has been definitively diagnosed. Administering hydration and sedation frequently, and bedrest are employed to halt preterm labor since these stop uterine activity by increasing intravascular volume and uterine blood flow.

The nurse identifies a nursing diagnosis of risk for injury related to possible effects of oxytocin therapy. Which action would the nurse perform to ensure a positive outcome for the client?

Assess contractions by using external monitor. In a client with the risk for injury, continuous assessment of contractions using external monitor and palpation to ensure the presence of a low resting tone will assist in collecting information about labor and the need for further intervention. Turning down oxytocin administration by half is required if hyperstimulation occurs not to prevent it. Tocolytic therapy is generally employed when preterm labor has been definitively diagnosed. Administering hydration and sedation frequently and bedrest are employed to halt preterm labor since these stop uterine activity by increasing intravascular volume and uterine blood flow.

A client's membranes have just ruptured. Her fetus is presenting breech. Which of the following should the nurse do immediately to rule out prolapse of the umbilical cord in this client? a) Place the woman in Trendelenburg position b) Administer oxygen at 10 L/min by face mask c) Administer amnioinfusion d) Assess fetal heart sounds

Assess fetal heart sounds

A client's membranes have just ruptured. Her fetus is presenting breech. Which of the following should the nurse do immediately to rule out prolapse of the umbilical cord in this client? a) Administer amnioinfusion b) Assess fetal heart sounds c) Administer oxygen at 10 L/min by face mask d) Place the woman in Trendelenburg position

Assess fetal heart sounds Explanation: To rule out cord prolapse, always assess fetal heart sounds immediately after rupture of the membranes whether this occurs spontaneously or by amniotomy, as the fetal heart rate will be unusually slow or a variable deceleration pattern will become apparent when cord prolapse has occurred. The other answers refer to therapeutic interventions to implement once cord prolapse has been confirmed.

A client's membranes have just ruptured. Her fetus is presenting breech. Which action should the nurse do immediately to rule out prolapse of the umbilical cord in this client?

Assess fetal heart sounds. To rule out cord prolapse, always assess fetal heart sounds immediately after rupture of the membranes whether this occurs spontaneously or by amniotomy, as the fetal heart rate will be unusually slow or a variable deceleration pattern will become apparent when cord prolapse has occurred. The other answers refer to therapeutic interventions to implement once cord prolapse has been confirmed.

Which postoperative intervention should a nurse perform when caring for a client who has undergone a cesarean birth?

Assess uterine tone to determine fundal firmness. When caring for a client who has undergone a cesarean birth, the nurse should assess the client's uterine tone to determine fundal firmness. The nurse should assist with breastfeeding initiation and offer continued support. The nurse can also suggest alternate positioning techniques to reduce incisional discomfort while breastfeeding. Delaying breastfeeding may not be required. The nurse should encourage the client to cough, perform deep-breathing exercises, and use the incentive spirometer every 2 hours. The nurse should assist the client with early ambulation to prevent respiratory and cardiovascular problems.

A client who is in labor presents with shoulder dystocia of the fetus. Which of the following is an important nursing intervention? a) Assess for prolonged second stage of labor with arrest of descent. b) Anticipate possible use of forceps to rotate to anterior position at birth. c) Assist with positioning the woman in squatting position. d) Assess for complaints of intense back pain in first stage of labor.

Assist with positioning the woman in squatting position.

A nurse is providing care to a couple who have experienced intrauterine fetal demise. Which action would be least effective in assisting a couple at this time?

Avoid any discussion of the situation with the couple. The nurse should encourage discussion of the loss and allow the couple to vent their feelings of grief and guilt. The nurse should allow the parents to spend unlimited time with their stillborn infant so that they can validate the death. Providing the parents and family with mementos of the infant helps validate the reality of the death. Assisting the family with arrangements is helpful to reduce the stress of coping with the situation and making decisions at this difficult time.

A client is admitted to the health care facility with a gestational age of 42 weeks. The client is to undergo a cesarean section. Which of the following would be the fetal risk associated with postterm pregnancy? a) Cephalopelvic disproportion b) Intraventricular hemorrhage c) Underdeveloped suck reflex d) Congenital heart defects

Cephalopelvic disproportion

A woman at 32 weeks' gestation is admitted in preterm labor. On your admission assessment, which of following findings should cause the nurse to question the administration of a tocolytic agent? a) Fetus in a breech presentation. b) Cervical dilation of 5 cm. c) Strong, regular contractions. d) A spontaneous abortion in an earlier pregnancy.

Cervical dilation of 5 cm.

A full-term pregnant client is being assessed for induction of labor. Her Bishop score is less than 6. Which of the following does it indicate? a) A cesarean birth may be required. b) Labor will occur spontaneously. c) Vaginal birth will be successful. d) Cervical ripening method should be used.

Cervical ripening method should be used.

A 16-year-old client has been in the active phase of labor for 14 hours. An ultrasound reveals that the likely cause of delay in dilatation is cephalopelvic disproportion. Which of the following interventions should the nurse most expect in this case? a) Administration of morphine sulfate b) Cesarean birth c) Administration of oxytocin d) Darkening room lights and decreasing noise and stimulation

Cesarean birth

A 16-year-old client has been in the active phase of labor for 14 hours. An ultrasound reveals that the likely cause of delay in dilatation is cephalopelvic disproportion. Which of the following interventions should the nurse most expect in this case? a) Administration of oxytocin b) Cesarean birth c) Darkening room lights and decreasing noise and stimulation d) Administration of morphine sulfate

Cesarean birth Correct Explanation: If the cause of the delay in dilatation is fetal malposition or cephalopelvic disproportion (CPD), cesarean birth may be necessary. Oxytocin would be administered to augment labor only if CPD were ruled out. Administration of morphine sulfate (an analgesic) and darkening room lights and decreasing noise and stimulation are used in the management of a prolonged latent phase caused by hypertonic contractions. These measures would not help in the case of CPD.

Before calling the physician to notify him or her of a slow progression or an arrest of labor several assessments need to be made. What other maternal assessment do you need to make prior to calling the physician? a) Make sure the patient is lying on their left side. b) Make sure the epidural medication is turned down. c) Assess vital signs every 30 minutes. d) Check for a full bladder.

Check for a full bladder.

Before calling the primary care provider to notify him or her of a slow progression or an arrest of labor, several assessments need to be made. What other maternal assessment does the nurse need to make prior to calling the care provider?

Check for a full bladder. A full bladder can interfere with the progress of labor, so the nurse must be sure that the client has emptied her bladder.

Before calling the physician to notify him or her of a slow progression or an arrest of labor several assessments need to be made. What other maternal assessment do you need to make prior to calling the physician? a) Make sure the epidural medication is turned down. b) Check for a full bladder. c) Make sure the patient is lying on their left side. d) Assess vital signs every 30 minutes.

Check for a full bladder. Correct Explanation: Remember that a full bladder can interfere with the progress of labor. So be sure that the patient has emptied her bladder.

A pregnant woman comes to the birthing center, stating she is in labor and doesn't know far along her pregnancy is because she has not had prenatal care. A physician performs an ultrasound that indicates oligohydramnios. When the patient's membranes rupture, meconium is in the amniotic fluid. What does the nurse suspect may be occurring with this patient? a) Complications of preterm labor b) Placental abruption c) Complications of placenta previa d) Complications of a post-term pregnancy

Complications of a post-term pregnancy

A pregnant woman comes to the birthing center, stating she is in labor and doesn't know far along her pregnancy is because she has not had prenatal care. A physician performs an ultrasound that indicates oligohydramnios. When the patient's membranes rupture, meconium is in the amniotic fluid. What does the nurse suspect may be occurring with this patient? a) Placental abruption b) Complications of a post-term pregnancy c) Complications of placenta previa d) Complications of preterm labor

Complications of a post-term pregnancy Correct Explanation: A post-term pregnancy carries risks for increased perinatal mortality, particularly during labor. Oligohydramnios and meconium staining of the amniotic fluid are common complications. Oligohydramnios increases the incidence of cord compression, which can lead to fetal distress during labor. Thick, meconium-stained fluid increases the risk for meconium aspiration syndrome.

Which of the following interventions would be most important when caring for the client with breech presentation confirmed by ultrasound? a) Continuing to monitor maternal and fetal status b) Applying suprapubic pressure against the fetal back c) Noting the space at the maternal umbilicus d) Auscultating the fetal heart rate at the level of the umbilicus

Continuing to monitor maternal and fetal status

Which of the following interventions would be most important when caring for the client with breech presentation confirmed by ultrasound? a) Continuing to monitor maternal and fetal status b) Auscultating the fetal heart rate at the level of the umbilicus c) Noting the space at the maternal umbilicus d) Applying suprapubic pressure against the fetal back

Continuing to monitor maternal and fetal status Correct Explanation: Once a breech presentation is confirmed by ultrasound, the nurse should continue to monitor the maternal and fetal status when the team makes decisions about the method of birth. The nurse usually plays an important role in communicating information during this time. Applying suprapubic pressure against the fetal back is the nursing intervention for shoulder dystocia and may not be required for breech presentation. Noting the space or dip at the maternal umbilicus and auscultating the fetal heart rate at the umbilicus level are assessments related to occipitoposterior positioning of the fetus.

A nurse is working with a client who has just begun labor and who has given birth vaginally five previous times. Which of the following interventions will the nurse most likely need to implement to meet the needs of this particular client?

Convert the birthing room to birth readiness before full dilatation is obtained Both grand multiparas (women who have given birth five or more times) and women with histories of precipitate labor should have the birthing room converted to birth readiness before full dilatation is obtained. Then, even if a sudden birth should occur, it can be accomplished in a controlled surrounding. As the client is likely to give birth relatively quickly, there is no need for oxytocin or to darken the room lights. There is also no indication that cesarean birth will be necessary, particularly because all of the client's previous births were vaginal.

A woman is admitted to the labor suite with contractions every five minutes lasting one minute. She is post-term and has oligohydramnios. What does this increase the risk of during delivery? a) Shoulder dystocia b) Cord compression c) Macrosomia d) Fetal hydrocephalus

Cord compression

A woman is admitted to the labor suite with contractions every five minutes lasting one minute. She is post-term and has oligohydramnios. What does this increase the risk of during delivery? a) Cord compression b) Macrosomia c) Shoulder dystocia d) Fetal hydrocephalus

Cord compression Explanation: Oligohydramnios and meconium staining of the amniotic fluid are common complications of post-term pregnancy. Oligohydramnios increases the incidence of cord compression, which can lead to fetal distress during labor.

A nurse working with a woman in preterm labor receives a telephone report for the fetal fibronectin test done 10 hours ago. The report indicates an absence of the protein, which the nurse knows indicates: a) Delivery is likely within the next 2 weeks. b) Delivery is unlikely within the 2 next weeks. c) Infection is present. d) No infection is present.

Delivery is unlikely within the 2 next weeks.

A woman's nurse-midwife tells her that the woman has developed dystocia. You would explain that this term means a) High blood pressure related to difficult labor. b) Muscle weakness related to prolonged labor. c) Potential for placental detachment. d) Difficult or abnormal labor.

Difficult or abnormal labor.

After an hour of administering oxytocin intravenously, you assess a woman's contractions to be 80 seconds in length. Your first action would be to a) Discontinue the oxytocin infusion. b) Increase the flow rate of the main line infusion. c) Slow the infusion to under 10 gtts per minute. d) Continue to monitor contraction duration every 2 hours.

Discontinue the oxytocin infusion. Explanation: If uterine contractions lengthen beyond 70 seconds, there is apt to be an interference with fetal circulation. Discontinuing the infusion allows contractions to shorten in length and allow fetal nourishment. You would not increase the flow rate of the main line infusion or slow the infusion without the physician's order. Uterine contractions are monitored continuously.

You assess that the fetus of a woman is in an occiput posterior position. Which of the following identifies the way you would expect her labor to differ from others? a) Need to have the baby manually rotated. b) Necessity for vacuum extraction for delivery. c) Shorter dilatational stage of labor. d) Experience of additional back pain.

Experience of additional back pain. Correct Explanation: Most women whose fetus is in a posterior position experience back pain while in labor. Pressure against the back by a support person often reduces this type of pain. An occiput posterior position does not make for a shorter dilatational stage of labor, it does not indicate the need to have the baby manually rotated, and it does not indicate a necessity for a vacuum extraction delivery.

A client in week 38 of her pregnancy has an ultrasound performed at a routine office visit and learns that her fetus has not moved out of a breech position. Which intervention does the nurse anticipate for this client? a) Forceps birth b) External cephalic version c) Vacuum extraction d) Trial labor

External cephalic version

A client in week 38 of her pregnancy has an ultrasound performed at a routine office visit and learns that her fetus has not moved out of a breech position. Which intervention does the nurse anticipate for this client? a) Vacuum extraction b) External cephalic version c) Trial labor d) Forceps birth

External cephalic version Correct Explanation: External cephalic version is the turning of a fetus from a breech to a cephalic position before birth. It may be done as early as 34 to 35 weeks, although the usual time is 37 to 38 weeks of pregnancy. A trial birth is performed when a woman has a borderline (just adequate) inlet measurement and the fetal lie and position are good and involves allowing labor to take its normal course as long as descent of the presenting part and dilatation of the cervix continue to occur. Forceps, which are not commonly used anymore, and vacuum extraction are used to facilitate birth when other complications are present, but would be less likely to be used with a fetus in breech position.

A placenta succenturiate is a placenta in which the cord is inserted marginally rather than centrally. a) True b) False

False

A placenta succenturiate is a placenta in which the cord is inserted marginally rather than centrally. a) False b) True

False Correct Explanation: In a battledore placenta, the cord is inserted marginally rather than centrally. A placenta succenturiata is a placenta that has one or more accessory lobes connected to the main placenta by blood vessels.

Tocolytic therapy will help to prevent preterm birth. a) False b) True

False Explanation: Tocolytic therapy does not typically prevent preterm birth, but instead it may delay it.

The client is 35 weeks of gestation and is being admitted for vaginal bleeding. The patient is stable at the time of admission. The priority nursing assessment for the client is for: a) Fetal heart tones. b) Signs of shock. c) Infection. d) Uterine stabilization

Fetal heart tones. Explanation: When a patient is admitted for vaginal bleeding and is stable, the next priority assessment is to determine if the fetus is viable. Options C and D are not a higher priority than fetal heart tones.

A nurse is caring for a pregnant client whose fetus has been diagnosed with macrosomia. When reviewing the client's history, which of the following would the nurse expect to find? a) Pre-term pregnancy b) Gestational diabetes c) Maternal rickets d) Small body size of mother

Gestational diabetes Correct Explanation: Macrosomia usually results from uncontrolled gestational diabetes, genetic problems, multiparity, or post-term pregnancy. Pre-term pregnancy, small body size of mother, and maternal rickets are not associated with macrosomia. Small body size and maternal rickets are associated with pelvic contraction at the inlet.

A nurse is assigned to care for a client who has been diagnosed with placental abruption. The nurse knows that which of the following could have led to placental abruption in the client? a) Gestational diabetes b) Gestational hypertension c) Cardiovascular disease (CVD) d) Obesity or excess weight gain

Gestational hypertension

At 31 weeks' gestation, a 37-year-old woman who has a history of preterm birth reports cramps, vaginal pain, and low, dull backache accompanied by vaginal discharge and bleeding. Her cervix is 2.1 cm long; she has fetal fibronectin in her cervical secretions, and her cervix is dilated 3 to 4 cm. For what do you prepare her? a) Careful monitoring of fetal kick counts b) Bed rest and hydration at home c) An emergency cesarean section d) Hospitalization, tocolytic therapy, and IM corticosteroids

Hospitalization, tocolytic therapy, and IM corticosteroids

A client has been in labor for 10 hours, with contractions occurring consistently about 5 minutes apart. The resting tone of the uterus remains at about 9 mm Hg, and the strength of the contractions averages 21 mm Hg. The nurse recognizes which of the following conditions in this client? a) Uncoordinated contractions b) Hypotonic contractions c) Braxton Hicks contractions d) Hypertonic contractions

Hypotonic contractions Correct Explanation: With hypotonic uterine contractions, the number of contractions is unusually infrequent (not more than two or three occurring in a 10-minute period). The resting tone of the uterus remains less than 10 mm Hg, and the strength of contractions does not rise above 25 mm Hg. Hypertonic uterine contractions are marked by an increase in resting tone to more than 15 mm Hg. However, the intensity of the contraction may be no stronger than that associated with hypotonic contractions. In contrast to hypotonic contractions, these occur frequently and are most commonly seen in the latent phase of labor. Uncoordinated contractions can occur so closely together they can interfere with the blood supply to the placenta. Because they occur so erratically such as one on top of another and then a long period without any, it may be difficult for a woman to rest between contractions or to breath effectively with contractions. Braxton Hicks contractions are sporadic contractions that occur in pregnancy before the onset of true labor.

A pregnant woman has just found out that she is having twin girls. She asks the nurse the difference between fraternal and identical twins. The nurse explains that with one set of twins there is fertilization of two ova, and with the other set one fertilized ovum splits. What type of twins result from the split ovum? a) both types can result from the split ovum b) fraternal c) neither type results from a split ovum d) identical

Identical Correct Explanation: The incidence of twins is about 1 in 30 conceptions, with about 2/3 being from the fertilization of two ova (fraternal) and about 1/3 from the splitting of one fertilized ovum (identical).

A physician orders oral tocolytic therapy for a woman with preterm labor. Which agent would the nurse be least likely to administer? a) Terbutaline b) Nifedipine c) Indomethacin d) Magnesium sulfate

Magnesium sulfate Correct Explanation: Magnesium sulfate is only given intravenously for preterm labor. Nifedipine and indomethacin are given orally for preterm labor. Terbutaline is given intravenously during the initial period and then switched to the oral route for maintenance.

You are assisting with delivery of the second child of a healthy young woman. Her pregnancy has been uneventful, and labor has been progressing well. The fetal head begins to deliver but instead of continuing to emerge, it retracts into the vagina. What should you try first? a) Attempt to push one of the fetus' shoulders in a clockwise or counterclockwise motion. b) McRobert's maneuver c) Apply pressure to the fundus. d) Zavanelli's maneuver

McRobert's maneuver Correct Explanation: This intervention is used with a large baby who may have shoulder dystocia and require assistance. The legs are sharply flexed, by a support person or nurse, and the movement will help to open the pelvis to the widest diameter possible. Zavanelli maneuver is performed when the practitioner pushes the fetal head back in the birth canal and performs an emergency cesarean delivery. Fundal pressure is contraindicated with shoulder dystocia. It is out of the province of the LVN to attempt delivery of the fetus by pushing one of the fetus' shoulders in a clockwise or counterclockwise motion.

Shoulder dystocia is a true medical emergency that can cause fetal demise because the baby cannot be delivered. Stuck in the birth canal, the infant cannot take its first breath. What is the first maneuver tried to deliver an infant with shoulder dystocia? a) McRonald Maneuver b) McRoberts maneuver c) McGeorge maneuver d) McDonald maneuver

McRoberts maneuver Correct Explanation: McRoberts maneuver is an intervention that is frequently successful in cases of shoulder dystocia, and is often tried first. McRoberts requires the assistance of two individuals. Two nurses are ideal; however, a support person or a technician can serve as the second assistant. With the woman in lithotomy position, each nurse holds one leg and sharply flexes the leg toward the woman's shoulders. This opens the pelvis to its widest diameters and allows the anterior shoulder to deliver in almost half of the cases.

A 26-year-old primigravida has brought her doula to the birthing center for support during her labor and delivery. The doula has been helping her through the past 16 hours of labor. The laboring woman is now 6 cm. dilated. She continues to report severe pain in her back with each contraction. The patient finds it comforting when her doula uses the ball of her hand to put counterpressure on her lower back. What is the likely cause of the woman's back pain? a) Occiput posterior position b) Fetal macrosomia c) Nongynecoid pelvis d) Breech presentation

Occiput posterior position

A 26-year-old primigravida has brought her doula to the birthing center for support during her labor and delivery. The doula has been helping her through the past 16 hours of labor. The laboring woman is now 6 cm. dilated. She continues to report severe pain in her back with each contraction. The patient finds it comforting when her doula uses the ball of her hand to put counterpressure on her lower back. What is the likely cause of the woman's back pain? a) Breech presentation b) Occiput posterior position c) Nongynecoid pelvis d) Fetal macrosomia

Occiput posterior position Correct Explanation: A labor complicated by occiput posterior position is usually prolonged and characterized by maternal perception of increased intensity of back discomfort. The lay term for this type of labor is "back labor."

The nurse is monitoring a client in labor who has had a previous cesarean section and is trying a vaginal birth with an epidural. The nurse observes a sudden drop in blood pressure, increased heart rate, and deep variable deceleration on the fetal monitor. The client reports severe pain in her abdomen and shoulder. What should the nurse prepare to do?

Prepare the client for a cesarean birth. The findings are consistent with uterine rupture. An abrupt change in the fetal heart rate pattern is often the most significant finding associated with uterine rupture. Others are reports of pain in the abdomen, shoulder, or back in a laboring woman who had previous good pain relief from epidural anesthesia. Falling blood pressure and rising pulse may be associated with hypovolemia caused by occult bleeding. The treatment is immediate cesarean birth.

The nurse is admitting the patient in labor. The physician determines that the fetus is in a transverse lie and not responsive to Leopold's maneuvers. What intervention should the nurse provide for the patient? a) Prepare to assist the physician with an amniotomy. b) Prepare for a precipitous vaginal birth. c) Administer an analgesic to the patient. d) Prepare the patient for a Cesarean section.

Prepare the patient for a Cesarean section.

The nurse is monitoring a patient in labor who has had a previous cesarean section and is trying a vaginal birth with epidural. The nurse observes a sudden drop in blood pressure, increased heart rate, and deep variable deceleration on the fetal monitor. The patient reports severe pain in her abdomen and shoulder. What should the nurse prepare to do? a) Prepare the patient for a cesarean section. b) Turn the patient on her left side. c) Bolus the patient with another dose of medication through the epidural. d) Place the patient in a knee-chest position.

Prepare the patient for a cesarean section.

The nurse is monitoring a patient in labor who has had a previous cesarean section and is trying a vaginal birth with epidural. The nurse observes a sudden drop in blood pressure, increased heart rate, and deep variable deceleration on the fetal monitor. The patient reports severe pain in her abdomen and shoulder. What should the nurse prepare to do? a) Turn the patient on her left side. b) Place the patient in a knee-chest position. c) Bolus the patient with another dose of medication through the epidural. d) Prepare the patient for a cesarean section.

Prepare the patient for a cesarean section. Correct Explanation: The findings are consistent with uterine rupture. An abrupt change in the fetal heart rate pattern is often the most significant finding associated with uterine rupture. Others are reports of pain in the abdomen, shoulder, or back in a laboring woman who had previous good pain relief from epidural anesthesia. Falling blood pressure and rising pulse may be associated with hypovolemia caused by occult bleeding. The treatment is immediate cesarean delivery.

Which of the following would be most appropriate for the woman who experiences dysfunctional labor in the first stage of labor? a) Provide ongoing communication about what is happening. b) Tell her not to feel anxious or discouraged about what is happening. c) Hold all explanations until after the birth to conserve the woman's energy. d) Limit talking to things the woman asks questions about.

Provide ongoing communication about what is happening. Correct Explanation: Dysfunctional labor at any point is frustrating to women. Maintaining open lines of communication at least keeps the woman well informed about what is happening.

Which intervention would be most appropriate for the woman experiencing dystocia related to problems involving the psyche? a) Encouraging the woman to assume a hands-and-knees position b) Preparing the woman for an amniotomy c) Administering oxytocin d) Providing a comfortable environment with dim lighting

Providing a comfortable environment with dim lighting

A woman near term presents to the clinic highly agitated because her membranes have just ruptured and she felt something come out when they did. You are alone with her and notice that the umbilical cord is hanging out of the vagina. What should you do next? a) With the woman in lithotomy position, hold her legs and sharply flex them toward her shoulders. b) Put her in bed immediately, call for help, and hold the presenting part of the cord. c) Go find assistance to confirm that the cord is in the vagina. d) Prep the woman for a vaginal delivery.

Put her in bed immediately, call for help, and hold the presenting part of the cord.

A woman near term presents to the clinic highly agitated because her membranes have just ruptured and she felt something come out when they did. The nurse is alone with her and notices that the umbilical cord is hanging out of the vagina. What should the nurse do next?

Put her in bed immediately, call for help, and hold the presenting part of the cord. Umbilical cord prolapse occurs when the umbilical cord slips down in front of the presenting part, when the presenting part compresses the cord oxygen, and nutrients are cut off to the baby, and the baby is at risk of death. This is an emergency. When a prolapsed cord is evident the nurse does not put the woman in lithotomy position, and the nurse does not leave the woman. A vaginal birth is contraindicated in this situation.

Which action by the nurse would be least effective in assisting a couple who have experienced intrauterine fetal demise? a) Assist the family in making arrangements for their stillborn infant b) Refrain from discussing the situation with the couple c) Allow the couple to spend as much time as they want with their stillborn infant. d) Give the parents a lock of the infant's hair

Refrain from discussing the situation with the couple Correct Explanation: The nurse should encourage discussion of the loss and allow the couple to vent their feelings of grief and guilt. The nurse should allow the parents to spend unlimited time with their stillborn infant so that they can validate the death. Providing the parents and family with mementos of the infant helps validate the reality of the death. Assisting the family with arrangements is helpful to reduce the stress of coping with the situation and making decisions at this difficult time.

The nurse would prepare a client for amnioinfusion when which of the following occurs? a) Fetal presenting part fails to rotate fully and descend in the pelvis b) Maternal pushing is compromised due to anesthesia c) The fetus shows non-reassuring fetal heart rate patterns d) Severe variable decelerations are due to cord compression

Severe variable decelerations are due to cord compression Correct Explanation: Indications for amnioinfusion include severe variable decelerations resulting from cord compression, oligohydramnios (decreased amniotic fluid), postmaturity, preterm labor with rupture of the membranes, and thick meconium fluid. Failure of the fetal presenting part to rotate fully, descend in the pelvis, non-reassuring fetal heart rate patterns or acute pulmonary edema, and compromised maternal pushing sensations from anesthesia are indications for forceps-assisted birth, and not for amniofusion.

The nurse would prepare a client for amnioinfusion when which action occurs?

Severe variable decelerations occur and are due to cord compression. Indications for amnioinfusion include severe variable decelerations resulting from cord compression, oligohydramnios (decreased amniotic fluid), postmaturity, preterm labor with rupture of the membranes, and thick meconium fluid. Failure of the fetal presenting part to rotate fully, descend in the pelvis, abnormal fetal heart rate patterns or acute pulmonary edema, and compromised maternal pushing sensations from anesthesia are indications for forceps-assisted birth, and not for amniofusion.

The rationale for using a prostaglandin gel for a client prior to the induction of labor is to: a. Stimulate uterine contractions b. Numb cervical pain receptors c. Prevent cervical lacerations d. Soften and efface the cervix

Soften and efface the cervix

A 39-year-old multigravida with diabetes presents to the clinic at 32 weeks' gestation because she has not felt the fetus moving lately. FHR is absent; sonogram confirms that the fetus has died. The nurse's institution has a policy of taking photographs of such fetuses once they are born. The nurse informs the woman that pictures have been taken and asks her if she wants them; she angrily tells the nurse no, then bursts into tears. How should the nurse respond?

Tell her that the hospital will keep the photos for her in case she changes her mind. Emotional care of the woman is complex. The woman may need time to move through the stages of grief and the responses of grief vary from person to person. The mother may request the items later and they should be stored or kept for a year after the delivery. There is no need to apologize to the client. It would be inappropriate to console her with the fact that she has other children. It negates her feelings and is not supportive of the woman at this time.

A 39-year-old multigravida with diabetes presents to the clinic at 32 weeks' gestation because she has not felt the fetus moving lately. FHR is absent; sonogram confirms that the fetus has died. Your institution has a policy of taking photographs of such fetuses once they are delivered. You inform the woman that pictures have been taken and ask her if she wants them; she angrily tells you no, then bursts into tears. How should you respond? a) Apologize and tell her that the photos will be destroyed immediately. b) Console her with the fact that she has other children. c) Tell her that the hospital will keep the photos for her in case she changes her mind. d) Tell her that once she gets over her shock and grief, she will probably be happy to have the photos.

Tell her that the hospital will keep the photos for her in case she changes her mind. Correct Explanation: Emotional care of the woman is complex. The woman may need time to move through the stages of grief and the responses of grief vary from person to person. The mother may request the items later and they should be stored or kept for a year after the delivery. Option A is incorrect as there is no need to apologize to the patient. Option B is incorrect as it would be inappropriate to console her with the fact that she has other children. Option D is incorrect as it negates her feelings and is not supportive of the woman at this time.

Which of the following would be appropriate for the use of low forceps? a) The fetal scalp is visible at the introitus without spreading the labia b) The fetal head is engaged but the leading point of the skull is less than +2 c) The leading point of fetal skull is at or above station +2, not on the pelvic floor d) The fetal skull has reached the pelvic floor, with the fetal head at the perineum

The leading point of fetal skull is at or above station +2, not on the pelvic floor Correct Explanation: Low forceps are applied when the leading point of the fetal skull is at or above station +2 and not on the pelvic floor. Outlet forceps, and not low forceps, are applied when the fetal skull has reached the pelvic floor, with the fetal head at the perineum and the fetal scalp visible at the introitus without spreading the labia. Mid forceps, and not low forceps, are applied when the fetal head is engaged but the leading point of the skull is less than +2.

Which of the following describes why hypertonic contractions tend to become very painful? a) More than one contraction may begin at the same time, as receptor points in the myometrium act independently of each other. b) The myometrium becomes sensitive from the lack of relaxation and anoxia of uterine cells. c) The number of uterine contractions is very low or infrequent. d) There is an increase in the length of labor because so many contractions are needed to achieve cervical dilation.

The myometrium becomes sensitive from the lack of relaxation and anoxia of uterine cells.

Which of the following describes why hypertonic contractions tend to become very painful? a) More than one contraction may begin at the same time, as receptor points in the myometrium act independently of each other. b) The myometrium becomes sensitive from the lack of relaxation and anoxia of uterine cells. c) The number of uterine contractions is very low or infrequent. d) There is an increase in the length of labor because so many contractions are needed to achieve cervical dilation.

The myometrium becomes sensitive from the lack of relaxation and anoxia of uterine cells. Correct Explanation: Hypertonic contractions cause uterine cell anoxia, which is painful. Therefore options A, C, and D are incorrect.

Mrs. M. has been admitted to the delivery suite in labor. She has been in labor for 12 hours and is dilated to 4 cm. The physician notes that Mrs. M. is in hypotonic labor. What does this mean? a) The uterine contractions may or may not be regular, but the quantity or quality or strength is insufficient to dilate the cervix. b) The uterine contractions may or may not be regular, but the quantity or quality or strength is sufficient to dilate the cervix. c) The uterine contractions are regular, but the quantity or quality or strength is insufficient to dilate the cervix. d) The uterine contractions are irregular, but the quantity or quality or strength is insufficient to dilate the cervix.

The uterine contractions may or may not be regular, but the quantity or quality or strength is insufficient to dilate the cervix. Explanation: There are two types of uterine dysfunction: hypotonic and hypertonic. The most common is hypotonic dysfunction. This labor pattern manifests by uterine contractions that may or may not be regular, but the quantity or strength is insufficient to dilate the cervix

Mrs. M. has been admitted to the delivery suite in labor. She has been in labor for 12 hours and is dilated to 4 cm. The physician notes that Mrs. M. is in hypotonic labor. What does this mean? a) The uterine contractions may or may not be regular, but the quantity or quality or strength is insufficient to dilate the cervix. b) The uterine contractions may or may not be regular, but the quantity or quality or strength is sufficient to dilate the cervix. c) The uterine contractions are irregular, but the quantity or quality or strength is insufficient to dilate the cervix. d) The uterine contractions are regular, but the quantity or quality or strength is insufficient to dilate the cervix.

The uterine contractions may or may not be regular, but the quantity or quality or strength is sufficient to dilate the cervix.

A client with a pendulous abdomen and uterine fibroid tumors had just begun labor and arrived at the hospital. After examining the client, the physician informs the nurse that the fetus appears to be malpositioned in the uterus. Which fetal position or presentation should the nurse most expect in this woman? a) Occipitoposterior position b) Anterior fetal position c) Transverse lie d) Cephalic presentation

Transverse lie Correct Explanation: A transverse lie, in which the fetus is more horizontal than vertical, occurs in women with pendulous abdomens, with uterine fibroid tumors that obstruct the lower uterine segment, with contraction of the pelvic brim, with congenital abnormalities of the uterus, or with hydramnios. Anterior fetal position and cephalic presentation are normal conditions. Occipitoposterior position tends to occur in women with android, anthropoid, or contracted pelves.

Hypertonic labor is labor that is characterized by short, irregular contractions without complete relaxation of the uterine wall in between contractions. Hypertonic labor can be caused by an increased sensitivity to oxytocin. What would the nurse do for a client who is in hypertonic labor because of oxytocin augmentation?

Turn off the pitocin. Hypertonic labor may result from an increased sensitivity of uterine muscle to oxytocin induction or augmentation. Treatment for this iatrogenic cause of hypertonic labor is to decrease or shut off the oxytocin infusion.

The nurse is assisting with the birth of the second child of a healthy young woman. Her pregnancy has been uneventful, and labor has been progressing well. The fetal head begins to emerge, but instead of continuing to emerge, it retracts into the vagina. What should the nurse try first?

Use McRobert's maneuver. This intervention is used with a large baby who may have shoulder dystocia and require assistance. The legs are sharply flexed, by a support person or nurse, and the movement will help to open the pelvis to the widest diameter possible. Zavanelli's maneuver is performed when the practitioner pushes the fetal head back in the birth canal and performs an emergency cesarean birth. Fundal pressure is contraindicated with shoulder dystocia. It is out of the province of the LVN to attempt birth of the fetus by pushing one of the fetus' shoulders in a clockwise or counterclockwise motion.

A nurse is assessing a full-term patient in labor and determines the fetus is occiput posterior. The patient states that all her discomfort is in her lower back. What intervention can the nurse provide that will help alleviate this discomfort? a) Use a fist to apply counter pressure to the lower back. b) Apply a warm washcloth to the lower back. c) Have the physician administer a pudendal block. d) Place the patient supine with the head of bed elevated 30 degrees.

Use a fist to apply counter pressure to the lower back. Correct Explanation: Counter pressure applied to the lower back with a fisted hand sometimes helps the woman to cope with the "back labor" characteristic of the occiput posterior position.

A client in the first stage of labor is diagnosed with dystocia involving the powers of labor. Which of the following would the nurse identify as the problem? a) Contractions are insufficient to cause fetus descent b) Uterine contractions are too weak or uncoordinated c) Pelvis is either android type or platypelloid type d) Fetus is in a different position or presentation

Uterine contractions are too weak or uncoordinated Correct Explanation: When there are problems with the powers causing dystocia during the first stage of labor, the uterine contractions are too weak or uncoordinated to cause adequate cervical effacement and dilatation. Contractions are insufficient to cause fetal descent; the fetus being in a different position or presentation, and pelvis being either android type or platypelloid type are not the results of dystocia. During the second stage of labor, the nurse should observe if the contractions and the pushing are insufficient to cause descent of the fetus. A fetus that is in a different position or presentation is a problem with the passenger. A pelvis that is either android type or platypelloid type is a problem with the passageway and is not related to dystocia.

While in labor a woman with a prior history of cesarean birth complains of light-headedness and dizziness. The nurse assesses the patient and notes an increase in pulse and decrease in blood pressure from the vital signs 15 minutes prior. What might the nurse consider as a possible cause for the symptoms? a) Uterine rupture b) Umbilical cord compression c) Placentea previa d) Hypertonic uterus

Uterine rupture

A pregnant woman near term is brought to the emergency room because she's been in an automobile accident. She has sustained blunt trauma to her abdomen and has gone into labor. An epidural is started and labor is going as well as can be expected under the circumstances. Suddenly the woman complains of severe pain in her back and shoulder. What do you suspect? a) Placental abruption b) Breech presentation c) Uterine rupture d) Broken bones or torn muscles from the accident

Uterine rupture Explanation: Uterine rupture occurs when the uterus tears open, leaving the fetus and other uterine contents exposed to the peritoneal cavity. Traumatic rupture can occur in connection with a blunt trauma. Abrupt change in the fetal heart rate pattern is often the most significant sign associated with uterine rupture. Other signs are complaints of pain in the abdomen, shoulder, or back in a laboring woman who had previous good pain relief from epidural anesthesia. The scenario presented does not indicate broken bones or torn muscles from the accident, placental abruption, or a breech presentation of the fetus.

After teaching a review class to a group of perinatal nurses about various methods for cervical ripening, the nurse determines that the teaching was successful when the group identifies which method as surgical?

amniotomy Amniotomy is considered a surgical method of cervical ripening. Breast stimulation is considered a nonpharmacologic method for ripening the cervix. Laminaria is a hygroscopic dilator that mechanically causes cervical ripening. Prostaglandins are pharmacologic methods for cervical ripening.

A nursing student correctly identifies the problem of fetal buttocks instead of the head presenting first as which type of presentation?

breech presentation Breech presentation is when the fetal buttocks present first rather than the head. Face and brow presentation has complete extension of the fetal head. Brow presentation is when the fetal head is between full extension and full flexion so that the largest fetal skull diameter presents to the pelvis. Persistent occiput posterior position is the engagement of fetal head in the left or right occiputo-transverse position with the occiput rotating posteriorly rather than into the more favorable occiput anterior position. Normal presentation is head first or occiput anterior.

A nursing student correctly identifies the problem of fetal buttocks instead of the head presenting first as which of the following? a) face and brow presentation b) breech presentation c) normal presentation d) persistent occiput posterior presentation

breech presentation Correct Explanation: Breech presentation is when the fetal buttocks present first rather than the head. Face and brow presentation has complete extension of the fetal head. Brow presentation is when the fetal head is between full extension and full flexion so that the largest fetal skull diameter presents to the pelvis. Persistent occiput posterior position is the engagement of fetal head in the left or right occiputo-transverse position with the occiput rotating posteriorly rather than into the more favorable occiput anterior position. Normal presentation is head first or occiput anterior.

Which intervention would be most important when caring for the client with breech presentation confirmed by ultrasound?

continuing to monitor maternal and fetal status Once a breech presentation is confirmed by ultrasound, the nurse should continue to monitor the maternal and fetal status when the team makes decisions about the method of birth. The nurse usually plays an important role in communicating information during this time. Applying suprapubic pressure against the fetal back is the nursing intervention for shoulder dystocia and may not be required for breech presentation. Noting the space or dip at the maternal umbilicus and auscultating the fetal heart rate at the umbilicus level are assessments related to occipitoposterior positioning of the fetus.

A client is giving birth when shoulder dystocia occurs in the fetus. The nurse recognizes that which condition in the client is likely to increase the risk for shoulder dystocia?

diabetes Shoulder dystocia is most apt to occur in women with diabetes, in multiparas, and in postdate pregnancies. A pendulous abdomen is associated with the transverse lie fetal position not with shoulder dystocia.

A nurse is assessing a pregnant woman who has come to the clinic. The woman reports that she feels some heaviness in her thighs since yesterday. The nurse suspects that the woman may be experiencing preterm labor based on which additional assessment findings? Select all that apply.

dull low backache malodorous vaginal discharge dysuria Symptoms of preterm labor are often subtle and may include change or increase in vaginal discharge with mucus, water, or blood in it; pelvic pressure; low, dull backache; nausea, vomiting or diarrhea, and heaviness or aching in the thighs. Constipation is not known to be a sign of preterm labor. Preterm labor is assessed when there are more than six contractions per hour.

The nursing student doing a clinical rotation in labor and delivery has noticed numerous women expressing various emotions during labor. The student's preceptor informs the student that these emotions can lead to psychological stress, which in turn can cause which of the following complications? a) pulmonary emboli b) premature labor c) dystocia d) deep vein thrombosis

dystocia

A client in week 38 of her pregnancy has an ultrasound performed at a routine office visit and learns that her fetus has not moved out of a breech position. Which intervention does the nurse anticipate for this client?

external cephalic version External cephalic version is the turning of a fetus from a breech to a cephalic position before birth. It may be done as early as 34 to 35 weeks, although the usual time is 37 to 38 weeks of pregnancy. A trial birth is performed when a woman has a borderline (just adequate) inlet measurement and the fetal lie and position are good and involves allowing labor to take its normal course as long as descent of the presenting part and dilatation of the cervix continue to occur. Forceps, which are not commonly used anymore, and vacuum extraction are used to facilitate birth when other complications are present, but they would be less likely to be used with a fetus in breech position.

A pregnant woman has just found out that she is having twin girls. She asks the nurse the difference between fraternal and identical twins. The nurse explains that with one set of twins there is fertilization of two ova, and with the other set one fertilized ovum splits. What type of twins result from the split ovum? a) both types can result from the split ovum b) neither type results from a split ovum c) fraternal d) identical

identical

A pregnant woman has just found out that she is having twin girls. She asks the nurse the difference between fraternal and identical twins. The nurse explains that with one set of twins there is fertilization of two ova, and with the other set one fertilized ovum splits. What type of twins result from the split ovum?

identical The incidence of twins is about 1 in 30 conceptions, with about 2/3 being from the fertilization of two ova (fraternal) and about 1/3 from the splitting of one fertilized ovum (identical).

A nursing instructor is teaching students about fetal presentations during delivery. The most common cause for increased incidence of shoulder dystocia is: a) poor quality of prenatal care b) longer lengths of labor c) increasing birth weight d) increased number of overall pregnancies

increasing birth weight

A nursing instructor is teaching students about fetal presentations during delivery. The most common cause for increased incidence of shoulder dystocia is: a) increased number of overall pregnancies b) poor quality of prenatal care c) increasing birth weight d) longer lengths of labor

increasing birth weight Correct Explanation: Shoulder dystocia is the obstruction of fetal descent and birth by the axis of the fetal shoulders after the fetal head has been delivered. The incidence of shoulder dystocia is increasing because of increasing birth weights, with reports of it in up to 2% of vaginal births

A nurse is reading a journal article about cesarean births and the indications for them. Place the indications for cesarean birth below in the proper sequence from most frequent to least frequent. All options must be used.

labor dystocia abnormal fetal heart rate tracing fetal malpresentation multiple gestation suspected macrosomia The most common indications for primary cesarean births include, in order of frequency, labor dystocia, abnormal fetal heart rate tracing, fetal malpresentation, multiple gestation, and suspected macrosomia.

What terminology would the nurse use to document a newborn who weighs 4,000 (8.13 lbs) or more at birth? a) meconia b) hydrocephalus c) macrosomia d) microsomia

macrosomia Correct Explanation: Macrosomia, in which a newborn weighs 8.13 to 9.15 lb or more at birth, complicates approximately 10% of all pregnancies. Meconium is the first stool passed by a newborn. Hydrocephalus is a buildup of fluid inside the skull.

A nursing student correctly identifies the most desirable position to promote an easy delivery as which of the following? a) occiput anterior b) face and brow c) shoulder dystocia d) breech

occiput anterior Correct Explanation: Any presentation other than occiput anterior or a slight variation of the fetal position or size increases the probability of dystocia.

A 26-year-old primigravida has brought her doula to the birthing center for support during her labor and birth. The doula has been helping her through the past 16 hours of labor. The laboring woman is now 6 cm. dilated. She continues to report severe pain in her back with each contraction. The client finds it comforting when her doula uses the ball of her hand to put counterpressure on her lower back. What is the likely cause of the woman's back pain?

occiput posterior position A labor complicated by occiput posterior position is usually prolonged and characterized by maternal perception of increased intensity of back discomfort. The lay term for this type of labor is "back labor."

When a woman in labor has reached 8 cm dilation, you notice the fetal heat rate suddenly slows. On perineal inspection, you observe the fetal cord has prolapsed. Your first action would be to a) cover the exposed cord with a dry, sterile wrap. b) turn her to her left side. c) replace the cord with gentle pressure. d) place her in a knee-chest position.

place her in a knee-chest position.

The nurse preceptor explains that several factors are involved with the "powers" that can cause dystocia. She focuses on the dysfunction that occurs when the uterus contracts so frequently and with such intensity that a very rapid birth will take place. This is known as which of the following? a) hypertonic contractions b) precipitous labor c) hypotonic contractions d) none of the above

precipitous labor

The nurse preceptor explains that several factors are involved with the "powers" that can cause dystocia. She focuses on the dysfunction that occurs when the uterus contracts so frequently and with such intensity that a very rapid birth will take place. This is known as which term?

precipitous labor When the expulsive forces of the uterus become dysfunctional, the uterus may either never fully relax (hypertonic contractions) placing the fetus in jeopardy, or relax too much (hypotonic contractions), causing ineffective contractions. Another dysfunction can occur when the uterus contracts so frequently and with such intensity that a very rapid birth will take place (precipitous labor).

The nurse preceptor explains that several factors are involved with the "powers" that can cause dystocia. She focuses on the dysfunction that occurs when the uterus contracts so frequently and with such intensity that a very rapid birth will take place. This is known as which of the following? a) hypertonic contractions b) precipitous labor c) hypotonic contractions d) none of the above

precipitous labor Correct Explanation: When the expulsive forces of the uterus become dysfunctional, the uterus may either never fully relax (hypertonic contractions) placing the fetus in jeopardy, or relax too much (hypotonic contractions), causing ineffective contractions. Another dysfunction can occur when the uterus contracts so frequently and with such intensity that a very rapid birth will take place(precipitous labor).

A woman having contractions comes to the emergency department. She tells the nurse that she is at 34 weeks' gestation. The nurse examines her and finds that she is already effaced and dilated 2 cm. What is this woman demonstrating? a) preterm labor b) normal labor c) macrosomia d) dystocia

preterm labor Correct Explanation: Preterm labor is the occurrence of regular uterine contractions accompanied by cervical effacement and dilation before the end of the 37th week of gestation. It is not normal labor. Macrosomia is a large fetus. Dystocia is difficult or abnormal labor.

Labor dystocia is an abnormal progression of labor. It is the most common cause of primary cesarean birth. When is it most common for labor dystocia to occur?

second stage of labor Labor dystocia can occur in any stage of labor, although it occurs most commonly once the woman is in active labor or when she reaches the second stage of labor.

A woman you are caring for during labor is having contractions 2 minutes apart but rarely over 50 mm Hg in strength; the resting tone is high, 20 to 25 mm Hg. She asks what she can do to make contractions more effective. Your best response would be that a) her physician will order oxytocin to strengthen contractions. b) she needs to rest because her contractions are hypertonic. c) hypotonic contractions of this kind will strengthen by themselves. d) walking around will make her contractions more regular.

she needs to rest because her contractions are hypertonic. Explanation: These contractions appear to be hypertonic because of the high resting tone. Hypertonic contractions occur because the uterus is being overstimulated or erratically stimulated. Rest is effective in helping contractions become more productive.

While in labor a woman with a prior history of cesarean birth reports light-headedness and dizziness. The nurse assesses the client and notes an increase in pulse and decrease in blood pressure from the vital signs 15 minutes prior. What might the nurse consider as a possible cause for the symptoms?

uterine rupture The client with any prior history of uterus surgery is at increased risk for a uterine rupture. A falling blood pressure and increasing pulse is a sign of hemorrhage, and in this client a uterine rupture needs to be a first consideration. The scenario does not indicate a hypertonic uterus, a placenta previa, or umbilical cord compression.

A client in her 42nd week of pregnancy is undergoing a scheduled induction of labor based on consideration of which of the following factors? Select all that apply. a) Abnormal fetal presentation b) Gestational age c) Fetal size d) Cervical ripeness e) Complete placenta previa

• Gestational age • Fetal size • Cervical ripeness Correct Explanation: Factors that the care provider should consider when deciding if and when to induce labor include cervical ripeness, gestational age and fetal size, fetal pulmonary maturity, fetal ability to tolerate labor, uterine sensitivity to the proposed induced method, and maternal condition. The health care provider does not confirm abnormal fetal presentation and complete placenta previa when deciding to induce labor. Abnormal fetal presentation and complete placenta previa are considered contraindications to the induction of labor and not as positive factors.

A primigravida at 28 weeks' gestation comes to the clinic for a check-up. She tells the nurse that her mother delivered both of her children prematurely, and she is afraid that the same will happen to her. What can the nurse inform her about the risk factors associated with premature births? (Select all that apply.) a) Current multiple gestation pregnancy b) Uterine or cervical abnormalities c) History of previous preterm birth d) Large-for-gestational age fetus e) Previous Cesarean section

• History of previous preterm birth • Current multiple gestation pregnancy • Uterine or cervical abnormalities

The nurse is preparing to talk to a group of pregnant women about elective induction and why it is not highly recommended. Which of the following should she include in her presentation? (Check all that apply.) a) It significantly increases the risk of C-section birth. b) It significantly increases instrumented delivery. c) It significantly increases the use of epidural analgesia. d) It significantly increases the admissions to the neonatal ICU. e) It significantly increases the weight of the newborn.

• It significantly increases the risk of C-section birth. • It significantly increases instrumented delivery. • It significantly increases the use of epidural analgesia. • It significantly increases the admissions to the neonatal ICU. Explanation: Evidence is compelling that elective induction of labor significantly increases the risk of cesarean birth, instrumented delivery, use of epidural analgesia, and neonatal ICU admissions. Increased birth weight is not a factor.

A client in labor has been diagnosed with shoulder dystocia. Which of the following risk factors would the nurse expect to assess in the client? Select all that apply. a) Intrauterine growth restriction b) Post-term pregnancy c) Maternal diabetes d) Incompetent cervix e) Fetal macrosomia

• Maternal diabetes • Post-term pregnancy • Fetal macrosomia

A client in labor has been diagnosed with shoulder dystocia. Which of the following risk factors would the nurse expect to assess in the client? Select all that apply. a) Post-term pregnancy b) Incompetent cervix c) Fetal macrosomia d) Maternal diabetes e) Intrauterine growth restriction

• Post-term pregnancy • Fetal macrosomia • Maternal diabetes Correct Explanation: Risk factors of shoulder dystocia include maternal diabetes, maternal obesity, post-term pregnancy, fetal macrosomia, previous history of shoulder dystocia, and multiparity. Intrauterine growth restriction and incompetent cervix are not the risks associated with shoulder dystocia. Intrauterine growth restriction is one of the factors that increase the risk of a breech presentation. Incompetent cervix is a risk factor related to preterm labor.

The second-year nursing student taking an obstetrics course correctly attributes which of the following to the term dystocia? (Check all that apply.) a) Progress of labor deviates from normal. b) Labor progresses normally. c) Labor is slow. d) Labor is fast.

• Progress of labor deviates from normal. • Labor is slow.

The second-year nursing student taking an obstetrics course correctly attributes which of the following to the term dystocia? (Check all that apply.) a) Labor progresses normally. b) Progress of labor deviates from normal. c) Labor is fast. d) Labor is slow.

• Progress of labor deviates from normal. • Labor is slow. Explanation: Dystocia is said to exist when the progress of labor deviates from normal and is slow.

Which of the following would lead the nurse to suspect that a pregnant woman is experiencing an amniotic fluid embolism? Select all that apply. a) Tachycardia b) Sudden onset of respiratory distress c) Acute, continuous abdominal pain d) Hypotension e) Sudden onset of fetal distress

• Sudden onset of respiratory distress • Hypotension • Tachycardia Explanation: The woman with an amniotic fluid embolism commonly reports difficulty breathing. Other signs include hypotension, tachycardia, cyanosis, seizures, coagulation difficulties, and uterine atony with subsequent hemorrhage. A sudden onset of fetal distress and acute continuous abdominal pain is associated with uterine rupture.

A client is 2 weeks past her due date, and her physician is considering whether to induce labor. Which of the following conditions must be present before induction can take place? (Select all that apply.) a) The fetus is in a longitudinal lie b) Cephalopelvic disproportion is present c) Absence of eclampsia d) The cervix is ripe e) A presenting part is engaged f) Normal maternal blood pressure

• The fetus is in a longitudinal lie • The cervix is ripe • A presenting part is engaged Correct Explanation: Before induction of labor is begun in term and postterm pregnancies, the following conditions should be present: the fetus is in a longitudinal lie; the cervix is ripe, or ready for birth; a presenting part is engaged; there is no cephalopelvic disproportion; and the fetus is estimated to be mature by date (over 39 weeks) or demonstrated by a lecithin-sphingomyelin ratio or ultrasound biparietal diameter to rule out preterm birth. Normal maternal blood pressure and absence of eclampsia are not conditions required for induction; in fact, severe hypertension and eclampsia are conditions that may necessitate induction.

A nurse preceptor asks a student to list commonly used diagnostic tests for preterm labor risk assessment. Which of the following tests should the student include? (Select all that apply.) a) U/A b) arterial blood gases c) amniotic fluid analysis d) CBC e) thyroid level

• U/A • amniotic fluid analysis • CBC Correct Explanation: Commonly used diagnostic testing for preterm labor risk assessment includes a complete blood count, urinalysis, and an amniotic fluid analysis.

A nursing student correctly identifies the causes of labor dysfunction to include which of the following? (Select all that apply.) a) problems with access to health care b) problems with the mother's diet c) problems with the uterus d) problems with the fetus e) problems with finances

• problems with the uterus • problems with the fetus


संबंधित स्टडी सेट्स

Chapter 9: Building a New Venture Team

View Set

AP Environmental Science Unit 7 Exam Review

View Set

OB Chapter 31; The Infant and Family

View Set